Infringement under the Doctrine of Equivalents: There is life in Q3

Reading time: 13 minutes
DISCLOSURE NOTICE: Hepp Wenger Ryffel is involved in this matter on behalf of Mepha Pharma AG.

Case Nos. O2021_004, O2021_005 | Judgment of 20 April 2023 | ‘Deferasirox II’

HEADNOTES

Art. 229 para. 1 CPC
Proper nova
The decision of a Board of Appeal of the European Patent Office which also concerns the patent in suit is a genuinely novel fact within the meaning of Art. 229 (1) (a) of the Code of Civil Procedure.

Art. 8 CC, Art. 1 para. 2 PatA, Art. 56 EPC
Burden of proof
The burden of proving the technical effect(s) of the invention lies with the patent proprietor.

Art. 51 PatA, Art. 66 let. a PatA, Art. 69 EPC
Infringement by equivalent means
If a preferred value is mentioned in the patent and a range is claimed in the claim that includes this value, the skilled person assumes that the patent proprietor has waived protection for values outside the claimed range.

This judgment in litigation between Mepha and Novartis covers both Mepha’s revocation action with respect to EP 202 and EP 018 (O2021_004) and Novartis’ conterclaim for infringement of both patents (O2021_005) by Deferasirox-Mepha® (Swiss market authorization no. 67678).

Fe(III) chelate

Deferasirox is an iron chelating agent; it is frequently referred to simply as DFX. It is mainly used to reduce iron overload in patients who are receiving long-term blood transfusions in the treatment of chronic anemias.

The FPC had issued a PI in summary proceedings in December 2021; see this Blog here. This finding has been reversed with the present landmark ruling in main proceedings, by an enlarged panel of five judges: The judgment holds that both patents are not infringed, neither literally nor under the DoE. This is what hn ③ is all about.

But let’s take it one step at a time, walking through all headnotes.

Board of appeal decision is proper novuM

Hn ① relates to a procedural matter:

Novartis logo

Let me put this into context first. In litigation at the FPC, parties have two unrestricted opportunities to comment on the merits of the case and, in particular, to introduce new facts into the proceedings. Thereafter, new facts and evidence (nova) can only be introduced under the limited conditions of Art. 229 CPC.

According to the Supreme Court (BGE 146 III 416, r. 4.1; see this Blog here), filing of a new claim set is to be treated in the same way as the submission of other nova. In the underlying case of this Supreme Court judgment, the patentee had partially waived the patent in ex parte proceedings at the Swiss IPI (Art. 24 PatA, comparable to limitation proceedings under Art. 105a EPC at the EPO) only after the closure of the file in co-pending litigation, and was barred from introducing the limited claims into the civil proceedings. The Supreme Court considered the limited claims as an improper novum under Art. 229 para. 1 let. b CPC: Even though the limited claims came into existence only after closure of the file, they could have come into existence earlier, only depending on the decision of the patentee with respect to both timing and extent of the partial waiver.

The present judgment holds that the situation in inter partes opposition / appeal proceedings at the EPO is different. First of all, such proceedings are not initated by the patentee. Second, and even though it is the patentee who formulates the wording of all his auxiliary requests, it is difficult for him to predict which of the auxiliary requests (if any) will survive examination on the merits. Finally, the timing of such inter partes proceedings is determined to large extent by the conduct of the other parties and the procedural calendar of the Board of Appeal. In sum, the FPC holds that the patentee is completely deprived of control over the time and at least partially deprived of control over the version in which the patent is maintained.

Mepha logo

Accordingly, the FPC held that the Board of Appeal decision T 1057/21 to maintain EP 018 in amended form is to be considered as a proper novum under Art. 229 para. 1 let. a CPC.

Let’s have a closer look at T 1057/21. EP 018 was maintained based on auxiliary request #48(!) of 71 auxiliary requests in total. Auxiliary request #48 had been initially filed as auxiliary request #38 in first instance proceedings on 19 February 2021, more than a year before closure of the file in nullity proceedings O2021_004. Patentee re-shuffled his requests later, i.e. the Board of Appeal did not deal with 47 requests before it finally allowed auxiliary request #48. Rather, the patentee made this request his new main request in the oral proceedings before the Boards of Appeal on his own volition, and this new main request was found allowable.

Undoubtedly, it is an uncomfortable situation for patentees to coordinate national nullity proceedings (governed by the strict rules on nova after closure of the file under the CPC) with co-pending central opposition / appeal proceedings at the EPO. Which is why some countries rule out national nullity proceedings while EPO opposition / appeal proceedings are still pending (e.g. Germany; see § 81(2) PatA); not so in Switzerland. The FPC’s approach will surely be appreciated by patentees, if and when it is ultimately approved by the Supreme Court.

burden of proof for technical effects

Hn ② relates to the allocation of the burden of proof for the technical effect(s) of the invention. Now we know:

On a quick read, the hn might suggest that it is up to the patentee to actually prove the technical effect, while the plaintiff in invalidity proceedings can sit back and just grouse that the claimed technical effect has not been proven (yet). But beware: The hn only deals with the allocation of the burden of proof.  It does not say anything about the standard of evidence according to the lex fori. The regular standard of ‘full conviction’ applies in Switzerland, which according to settled case law means that the court should have no serious doubts about the existence of the alleged fact, or that remaining doubts appear to be slight.

Now, what does that mean for assessment of an alleged technical effect of an invention?

The proof does not necessarily have to be provided with empirical data or even clinical studies. If the person skilled in the art can conclude on the basis of scientific or theoretical considerations that the claimed technical effect exists, this is sufficient proof. When assessing the evidence, it may also be taken into account which indications speak against the claimed technical effect. If there are no such indications, the requirements for proving the technical effect are lower.

Tango
it takes two …

Not much of a surprise, isn’t it? In practice, it takes two to tango: The more indications are on the table that the effect is not achieved, the harder it will be for the patentee to actually prove the contrary. Even though the burden of proof lies with the patentee in the first place (which is also the case at the EPO, see e.g. T 1188/00 (hn) and G 2/21 (r. 26)), the whole discussion about whether or not a certain technical effect is achieved is an argumentative arm wrestling.

Numerical ranges / infringemement under the DOE

The judgment holds that both EP 202 and EP 018 as amended in opposition/appeal proceedings are valid (r. 46-67) but not infringed, neither literally nor under the DoE.

The decisive feature in terms of infringement is the amount of deferasirox. The independent claims of both patents require 45-60% by weight of deferasirox. The attacked embodiment contained 64.3% by weight, based on the information given in the product information; r. 68. Even the lowest (disputed) analytical value submitted by the patentee was 60.8%. The judgment holds that even in case of the lower value literal infringement is ruled out. The skilled person will typically consider numerical ranges as more precise and more clear than other verbally described technical claim features (in line with the judgments X ZR 168/00Schneidmesser I (r. II 3 lit. d) and X ZR 135/01Schneidmesser II (r. 4 lit. d) of the German Supreme Court). Accordingly, also values within usual tolerances are not literally covered. The skilled person knows, however, that technical numerical ranges are subject to a measuring error. Therefore, as a rule, the last digit of numerical values corresponds to the measurement accuracy. If no other error limits are indicated (as is the case here), the maximum error for the last digit is estimated from the rounding convention; see e.g. T 175/97 (r. 2.6). The judgment holds that this case law developed for prior art disclosures equally applies to the interpretation of numerical ranges in patent claims. Since the claimed range is given without any decimal digits, the range that is still within the literal scope of protection is ≥44.5% to <60.5% if the normal rounding convention is applied.

Accordingly, and in particular in the absence of any teaching in the patents that would point to a broader interpretation of the claimed range, the judgment holds that neither 64.3% nor 60.8% is literally covered by the claims.

Turning to infringement under the DoE (r. 73 et seqq.), the judgment holds that the variant has the same effect; i.e., Q1 of the three prong test is answered in the affirmative (not much out of the ordinary for generics).

As to the second question re accessibility of the variant (‘Auffindbarkeit’), the judgment holds that there is currently some uncertainty as to whether this should be dealt with in retrospect (i.e., in knowledge of the variant like in the U.K.), or not (like in Germany); r. 77. It is hoped that the Supreme Court clarifies its position when it gets the chance (which is now):

In the case at hand, the FPC held that it does not matter: The same effect of the variant was ‘accessible’ without involvement of an inventive step in any event; r. 78. Thus, Q2 is answered in the affirmative.

Q3 is meant to safeguard the legitimate interest of the public in legal certainty. Here it is (my own understanding / translation):

Does an objective reading of the patent specification lead the skilled person to the conclusion that the patentee has formulated the claim — for whatever reason — so narrowly that he does not claim protection for an embodiment of same effect (Q1) that is ‘accessible’ for the skilled person (Q2)?

The single embodiment of both patents makes use of 56% of deferasirox (para. [0016] von EP 202; para. [0017] von EP 018). In the claims, however, not this specific value is mentioned, but rather a range of 45% to 60%, which is evidently placed around this specific value. Under these circumstances, the judgment holds (r. 80) that the skilled person assumes that the patentee has specifically and bindingly placed the claimed range around the specific value, i.e. the patentee has deliberately decided to specify the invention to be within this range and thus to dispense with the scope of protection lying outside this range. Under the circumstance, the judgment holds that there is no room to extend this range under the DoE by taking into account generally accepted tolerances going beyond it, because the skilled person may in good faith assume that the patentee has already taken into account possible tolerances when defining the range. In passing, the judgment notes that the range is not symmetrically placed around the specific value; it rather is narrower above than below the specific value. This even underlines the importance of the upper limit. Thus, infringement under the DoE was denied.

This outcome is essentially in line with the U.K. judgment [2022] EWHC 2847 (Pat), para. 232 (emphasis added):

I have found that in the present case the inventive concept of the Patents would be seen by the skilled team as requiring strict compliance with the 45-60% range of deferasirox. On that basis a variant will not be substantially the same as the inventive concept unless it strictly complies. I need not investigate with precision what compliance means numerically as matter of general principle. In my judgment Teva DFX does not comply.

In the Swiss case, Novartis appealed to the Supreme Court. The appeal is still pending at the time of writing.

In accordance with the outcome in main proceedings, the PI was lifted with separate order.

UPDATE 28 Sept. 2023: Novartis’ appeal has been dismissed by the Supreme Court; the decision is final.

✍ MW

BIBLIOGRAPHY

Case Nos. O2021_004, O2021_005 | Judgment of 20 April 2023 | ‘Deferasirox II’

Mepha Pharma AG
./.
Novartis AG

Panel of Judges:

    • Dr. Mark SCHWEIZER
    • Dr. Tobias BREMI
    • Dr. Rudolf RENTSCH
    • Prisca von BALLLMOOS
    • Marco ZARDI

Judge-rapporteur:

    • Dr. Tobias BREMI

Court Clerk:

    • Sven BUCHER

Representative(s) of Plaintiff:

Representative(s) of Defendant:

JUDGMENT
of 20 April 2023 in main proceedings

ORDER
of 12 May 2023 rectifying / supplementing the judgment

JUDGMENT
of 15 December 2021 in summary proceedings

PATENTS IN SUIT

EP 2 964 202 B1:

See EPO Register and Swissreg for bibliographic details.

EP 2 964 202 B2:

See EPO Register and Swissreg for further information.

EP 3 124 018 B1:

See EPO Register and Swissreg for further information.

EP 3 124 018 B2:

Not yet available. Currently being dealt with by the OD again, after T 1057/21,  to compile the set of documents for maintenance of the patent in amended form, based on the following claims:

See EPO Register and Swissreg for further information.

EPO BOARD OF APPEAL DECISIONS

T 0526/21 re EP 202

T 1057/21 re EP 018

BE ON THE KNOW

Enter your name and email address below to get notified of new posts by email.

How many deaths can one die?

Reading time: 11 minutes

Case No. O2017_015 | Decision of 7 November 2019 | ‘Rame ferroviaire modulaire’

HEADNOTE

Art. 56 EPC, Art. 1(2) PatA: Inventive step and technical disadvantages in the event of a change in the teaching of the state of the art.

If an obvious technical disadvantage for the skilled person results from a change of the teaching of a prior art leading to the claimed invention, it cannot be asserted, that the skilled person would not seriously consider such a change of the teaching of the prior art, if at the same time the claimed invention does not remedy that disadvantage in an unexpected manner, but merely accepts it.

Alstom’s logo

The patent at stake in this infringement case is Alstom‘s EP 1 024 070 B1; see EPO Register and Swissreg for further bibliographic information. We have reported about the main hearing in this matter on this Blog here.

Alstom alleged that Bombardier‘s Twindexx train in the IR100 (SBB RABe 502) configuration infringes EP’070:

The IR100 configuration of Bombardier’s Twindexx train
SBB logo

Please find some further information about the Twindexx train on SBB’s website, and on Bombardier’s swissdosto.ch website and fact sheet. The video below is embedded from swissdosto.ch, for amusement of the railway enthusiasts out there before we are going to have a closer look at the more legal issues of the case …

Promo video, embedded from swissdosto.ch (Bombardier)

The relevant claims are the independent claim 1 and its dependent claim 7 (emphasis added), as asserted by Alstom:


Claim 1:

A modular railway rake formed of at least two vehicles (1, 2) with two superposed levels, including at least one bogie (10) including at least one driving axle, at least one bogie (11) including at least one carrying axle, and on-board equipment including at least one power supply electronic unit (19) for supplying motive power to one or more traction motors, at least one motive power supply unit (22, 25) for supplying power to the electronic unit (19), at least one auxiliary power supply unit (30) for supplying power to auxiliary equipment, and at least one device (21) for connection to an overhead power supply network, in which rake the motive power factor of a vehicle or a group of vehicles is defined as the ratio of the number of driving axles to the total number of axles of the vehicle or group, the rake being characterized in that at least one vehicle (1) includes at least one bogie (10) including at least one driving axle and at least one bogie (11) including at least one carrying axle, at least one bogie (11) of the rake including at least one carrying axle is associated with at least one of said on-board equipment items (19; 22, 25; 30; 21) disposed approximately over that bogie, and the ratio between the respective motive power factors of any two groups of two vehicles of the rake is not less than 1/3 and not more than 3.

Claim 7:

A railway rake according to claim 1, characterized in that at least one power supply unit (22, 25; 30) is disposed approximately over a bogie (11) including at least one carrying axle.
Bombardier Logo

Bombardier took the position that the IR100 did not make use of the features as marked-up in bold above; in particular, that the IR100 was not modular and did not have two superposed levels in the sense of EP’070.

Alstom had amended the request for injunctive relief after the instructional hearing in that the term ‘modular’ was replaced by the following:

What ‘modular’ means, in Alstom’s view

With regard to modularity, the court came to the conclusion that only a modularity could be meant which allows the train operator to react on momentary needs, by way of adding vehicles to preexisting train compositions. The decision refers to ¶¶[0002]-[0003] of the general specification of EP’070, as well as the explanations with respect to specific embodiments:

Alstom had argued that also modifications in medium and long term would be covered by the term ‘modular’ when properly construed. This, however, did not convince the judges because such broad reading would cover virtually any system composed of a plurality of parts. The term must necessarily have a (more narrow) meaning. Otherwise, so the decision, it would not have been used (¶19):

Par conséquent, étant donné que la rame ferroviaire est définie dans la revendication comme etant «modulaire», ce terme doit nécessairement avoir une signification particulière, sinon il n’aurauit pas été spécifié dans la revendication.

That’s pretty catchy, isn’t it? I would not be surprised if (mis?)use of this reasoning will become somewhat popular in response to an invalidity challenge:

Uuhu, hold on, the feature is in the claim, it must have a very specific meaning, else it wouldn’t be there.

Well, not necessarily. Sometimes there are features in a claim that are just trivial — and can be (easily) challenged as such.

Anyway, the decision holds that omission of the term ‘modular’ in the plaintiff’s request for injuctive relief as amended resulted in the court’s finding that the request went beyond the scope of protection of EP’070 and that it had to be rejected already for this reason alone; ¶21. Death #1.

This could have been it. However, the decision gives even more reasons as to why the complaint had to be rejected: EP’070 was neither valid nor infringed.

After a detailed claim construction, the decision holds that EP’070 to the invoked extent was novel over the closest prior art, D1, at least for the modularity of the rake and for the positioning of heavy elements approximatively above the carrying axle bogie.

Now … what is D1?!

But … what the heck is D1?

The decision is silent on what D1 is — which is regrettable. No chance to really review the stringency of the court’s line of arguments. It can only be assumed from what had been pleaded at the main hearing and some annotations in figures reproduced in the decision that D1 is a publication about the first generation DD-IRM (1994) train of the Dutch Railways (see Wikipedia: NS_VIRM).

Anyway, the decision holds that the subject-matter of EP’070 to the invoked extent did not involve an inventive step over the teachings of D1. The differentiating features (first, the modularity; and, second, the positioning of heavy elements approximatively above the carrying axle bogie) are associated with the technical effects that the weight is distributed evenly over the length of the train rake and that train operation becomes more profitable.

As to modularity, the decision holds that the skilled person would arrive at a modular train without further ado if he/she were to develop a more flexible and hence more profitable train rake.

With regard to the positioning of heavy equipment (such as the down chopper module in D1) the judges agreed with the patentee in that such heavy equipment would intuitively be placed in the lowest possible location of the rake, e.g. at the bottom of a low voltage machine room. However, and this is where the headnote stems from, if the claimed invention merely accepts an obvious disadvantage without suggesting any remedy thereto, the unfavorable feature cannot support the finding of an inventive step.

... when an apparent technical disadvantage of a technical solution is not overcome in a non-obvious manner but merely accepted.

In consequence, EP’070 was found obvious over D1 in combination with the skilled person’s general technical knowledge. Death #2.

At the face of it, the hn makes much sense against the background of the case at hand. But is the absolute character of the statement justified? One might well think of situations, where a particular disadvantage of a feature would be merely accepted (rather than remedied), but where the choice of the feature would still be non-obvious. For example, when the commonly known and persisting drawbacks are set off by other, unrelated or indirect advantages associated with the feature. In our perception, the hn should be understood with the provision marked-up below:

[…] if at the same time the claimed invention does not remedy that disadvantage in an unexpected manner, but merely accepts it and does not compensate for it by any unexpected technical advantage.

This would actually be perfectly in line with established case law of the Boards of Appeal of the EPO; see Case Law of the Boards of Appeal (2019), I.D.9.19.1, in particular T 119/82 (hn 2).

Again, the decision could have ended at this point. But the decision also expands on the question of infringement, i.e. that Bombardier’s Twindexx IR100 was not modular in the sense as described above. In particular, it could not, due to software issues, be supplemented with additional coaches nor could coaches be removed by the train operator in the short run. Also, no equivalent feature providing such flexibility could be found in the IR100 train. Death #3.

It is remarkable that the decision gives no less than three reasons for rejecting the request for injunctive relief. This is rarely seen. Still, it may well serve procedural economy: Both the parties and the appellate body are provided with a more complete picture of the first instance’s views. This way, the unsuccessful party can make a more informed decision on whether or not to lodge an appeal; and if an appeal is lodged, the appellate body might not have to remit the case back to the first instance — hence saving time and money.

Patent Reaper

But as if three reasons were not enough, death #4 was just around the corner: EP’070 lapsed after 20y in January 2020.

The decision has not been appealed / is final.

Reported by Jolanda MINDER and Martin WILMING

BIBLIOGRAPHY

Case No. O2017_015 | Decision of 7 November 2019 | ‘Rame ferroviaire modulaire’

Alstom Transport Technologies SASU
./.
Bombardier Transportation (Switzerland) AG

Panel of Judges:

    • Dr. Philippe DUCOR
    • Dr. Tobias BREMI
    • Christophe SAAM

Judge-rapporteur:

    • Dr. Tobias BREMI

Court Clerk:

    • Agnieszka TABERSKA

Representative(s) of Plaintiff:

Representative(s) of Defendant:

DECISION IN FULL

PATENT IN SUIT

BE ON THE KNOW

Enter your name and email address below to get notified of new posts by email.

Obviousness is a question of law – but the court will only pick up lines of attack that have been asserted

Case No. O2016_009 ¦ Decision of 18 December 2018 ¦ ‘Durchflussmessfühler’

Reading time: 18 minutes

HEADNOTE

Art. 226, 229 CPC
New facts, amendment of patent claims.

If in a patent infringement proceeding the plaintiff files before the instructional hearing a reply limited to the defense of the validity and no pleadings take place at the instructional hearing, the plaintiff may submit new allegations of fact, and therefore also amended patent claims, in the completed reply filed after the instructional hearing without having to meet the requirements of art. 229 para. 1 CPC.

Please find some background information about the subject-matter at stake on this Blog here, and a report about the main hearing of 29 October 2018 on this Blog here.

Vyaire logo

In brief, Hamilton Medical AG alleged infringement of its Swiss patent CH 701 755 B1 (see Swissreg for further bibliographic details) by some of imtmedical AG‘s iFlow sensors. Note that Vyaire has recently taken over imtmedical’s ventilator and related consumables business; see imtmedical’s press release here.

The only independent claim 1 of CH’755 as granted reads as follows:

Durchflussmessfühler (10) mit einem ein zylindrisches Gehäuse (10) definierenden Durchgang mit einer ersten Durchgangsöffnung (13) und einer zweiten Durchgangsöffnung (29); einem im Durchgang des Gehäuses angeordneten Strömungswiderstand (23), welcher das Gehäuse in einen ersten und einen zweiten Gehäuseteil (11 und 27) unterteilt; einer ersten Anschlussstelle (17) mit einer Verbindung zum Innern des ersten Gehäuseteils (11); und einer zweiten Anschlussstelle (19) mit einer Verbindung zum Innern des zweiten Gehäuseteils; dadurch gekennzeichnet, dass die ersten und zweiten Anschlussstellen (17, 19) in einem Abstand voneinander auf dem gleichen Gehäuseteil angeordnet sind.

Inofficially translated to English language:

Flow sensor (10) with a passage defining a cylindrical housing (10) with a first passage opening (13) and a second passage opening (29); a flow resistor (23) arranged in the passage of the housing which divides the housing into a first and a second housing part (11 and 27); a first terminal (17) having a connection to the interior of the first housing part (11); and a second terminal (19) having a connection to the interior of the second housing part; characterised in that the first and second terminals (17, 19) are spaced apart on the same housing part.

Fig. 1 of CH’755 surely helps to understand what the invention is about:

Fig. 1 of CH 701 755 B1

The defendant denied an infringement, and argued for nullity as a plea in defense.

The verbal limitation of the patent

This is what the hn is all about. Imagine a plaintiff / patentee in infringement proceedings who is confronted with a plea for nullity in defense: When is his last chance to formally assert his patent only to a more limited and presumably valid extent?

First, it is important to understand how the FPC conducts the proceedings. If the defendant raises a plea for nullity in his answer to plaintiff’s statement of claim in infringement proceedings, the FPC invites the plaintiff to file a reply which is strictly limited to only the nullity aspects of the case. What follows next is an instructional hearing. In preparation of that hearing, the limitation of plaintiff’s reply to only the nullity aspects is to assure that the court has a single exchange of opinions at hand for both infringement and nullity. If the case moves on after the instructional hearing, it is again up to the plaintiff to complete his reply. Thereafter, the defendant provides the rejoinder re both infringement and nullity. This is the end of the regular exchange of briefs in which the parties can present facts and arguments without the need to comply with Art. 229(1) CPC (novae). However, the plaintiff may of course comment on new allegations and facts presented by the defendant in his rejoinder — within a deadline set by the FPC (typically two weeks, extendable by one further week; see this Blog here), or under the unconditional right to be heard (within 10 days).

The below is an illustrative summary of this chain of events:

Plaintiff Defendant
Statement of Claim
re infringement
Answer
w nullity plea in defense
First reply
re alleged nullity only
Instructional hearing w/o pleadings
Second reply
re infringement and nullity
Rejoinder
re infringement and nullity
Comments
on new factual allegations in the rejoinder, if any
Judge rapporteur’s expert opinion

Note the subtitle to the second reply in the above: The decision holds that the plaintiff may present arguments and facts in his second reply after the instructional hearing not only relating to infringement aspects, but rather also to the nullity aspects of the case.

In the defendant’s view, the plaintiff thus had three chances to deal with validity / nullity of the patent, i.e. in his initial statement of claim and both his replies. The FPC disagreed: There is no reason for a plaintiff to deal with nullity in the beginning. How would he even do so in the absence of any knowledge of the defendant’s potential arguments? Thus, the FPC held that both parties had exchanged their views on both aspects of the case for the first time only after plaintiff’s first reply (limited to nullity). With the second reply and the rejoinder, both without limitations re novae, the double exchange of briefs is concluded.

Undoubtedly, an amended claim is to be considered a new fact (decision 4A_543/2017 of the Supreme Court, ¶2.3); as such, it would be an inadmissible novum after the normal exchange of briefs.

The plaintiff ‘verbally limited’ the claim only in his second reply. The defendant argued that this second reply was limited to only infringement aspects; it should only complement the first reply that had already dealt with nullity. Defendant argued that this was established practice at the FPC. Again, the FPC disagreed: There was not an established practice of the FPC in that sense.

now, what?!

Interestingly, the former President Dieter Brändle had apparently indicated in the instructional hearing that the patent could not be asserted in a more limited form any more after the instructional hearing. Defendant argued that it is against good faith if the limitation was now accepted nevertheless. However, the decision holds that it is up to the whole panel of judges to finally rule on admissibility of new facts; Mr. Brändle only gave his personal, preliminary opinion on this issue at that time. When the defendant had been invited to provide his rejoinder, this came along with an explicit note to deal with plaintiff’s reply as a whole, and that the whole panel of judges would finally decide on admissibility of the limitation of the patent. Thus, the defendant had no procedural disadvantage.

On a sidenote, I am wondering what the last point of time would be for a patentee / plaintiff to limit the claim when faced with a full-fledged counterclaim for nullity, i.e. when there are two parallel tracks of claim and counterclaim:

Plaintiff
Defendant
Statement of Claim
(re infringement)
Answer
re claim
Counterclaim
(re nullity)
Answer
re c’claim
Instructional hearing w/o pleadings
Reply
re claim
 
Rejoinder
re claim
Reply
re c’claim
Rejoinder
re c’claim
  Comments
on new factual allegations in the rejoinder re c’claim, if any
Judge rapporteur’s expert opinion

Would the plaintiff’s reply after the instructional hearing then be limited to the infringement part, and the final chance to limit the claim be the rejoinder re c’claim? This remains to be seen in a case to come.

General remarks on claim construction

The decision summarises certain key aspects of claim construction in ¶25 (and, later on, in ¶47). I do readily agree with most of it, but stumbled upon the following:

When case-law refers to ‘broadest interpretation’ of claim features,15 the feature must still be capable of fulfilling its purpose in the context of the invention. This means that the claim must not be interpreted under its wording, but also not in such a way that embodiments are covered which do not achieve the inventive effect.

15 FPC, decision O2013_008 of 25 August 2015, ¶4.2 — ‘elektrostatische Pulversprühpistole’ [see this Blog here]

The decision cites ample case-law, but there is no fn in the last sentence. Now, where does that come from? The sentence somehow reminds me of hn 3 of the ‘Spannschraube’ decision of the German Federal Supreme Court, X ZR 85/96 of 2 March 1999. The German Supreme Court had held:

The scope of protection of a European patent cannot be extended to embodiments which wholly (or to an extent which is practically insignificant) waive the success sought by the patent.

However, the Swiss Supreme Court had to assess the same subject-matter shortly thereafter, and explicitly disagreed; 4C.348/1999 of 12 April 2001: What had been considered to be no infringement in Germany, was well an infringement in Switzerland!

The Swiss Supreme Court did not see any reason to deviate from the literal and technically plausible meaning of the feature — even though an explicit object of the patent that had been recited in the specification was not achieved by such embodiments, i.e.  the use of very short clamping screws (see EP 0 319 521 B1, col. 1, l. 54-57):

Der Erfindung liegt die Aufgabe zugrunde, eine Rohrschelle der eingangs genannten Art zu schaffen, die sich auch mit einer sehr kurzen Spannschraube gut handhaben und leicht schliessen lässt.

It remains to be seen whether the present decision might indicate a gradual departure from the old Spannschrauben approach of the Swiss Supreme Court.

Alleged nullity 

The claims at stake can be summarized as follows, structured into features:

a) Flow sensor (10) with
b) a passage defining a cylindrical housing (10) having a first through opening (13) and a second through opening (29);
c) a flow resistor (23) arranged in the passage of the housing, which divides the housing into a first and a second housing part (11 and 27);
d) a first connection point (17) having a connection to the interior of the first housing part (11);
e) a second connection point (19) having a connection to the interior of the second housing part;
f) the first and second connection points (17, 19) being arranged at a distance from one another on the same housing part;
i) claim 4 as granted:
wherein flanges are provided on the first housing part (11) and on the second housing part (27) for connecting the housing parts with flange pressure surfaces (15, 16, 31, 32) are formed;
j) first variant of claim 10 as granted:
wherein the connections run partly in the flange pressure surfaces,
and/or
k) second variant of claim 10 as granted:
[wherein the connections] exit through the flange pressure surface as openings on the sealing surface side (17, 19, 21, 33, 35, 37) .

Features a) – f) correspond to claim 1 as granted (marked in bold in the above); the whole set of features, i.e. a) – f) and j) – k), corresponds to the claim as limited with plaintiff’s second reply. Don’t get confused by the missing features g) and h): The parties referred to these features in their feature analysis; but they are irrelevant for the present decision since they only relate to dependent claims 2 and 3 which are not at stake.

The decision holds that claim 1 as granted lacks novelty over document E10, i.e. (JP S61-205023); see Fig. 1:

JP S61-205023, Fig. 1

Next, the decision assesses and acknowledges novelty of the more limited claim over E1 (US 6,585,662 B1) which lacks features j) and k). Defendant had also argued that a prior use of the device of E1 had occured (prior use ‘Jones’). However, the decision holds that this prior use does not need to be assessed any further: Defendant had argued that the prior use disclosed a device according to the teaching of E1. Thus, the decision holds that any such prior use would necessarily also have lacked features j) and k).

Novelty over E10 (JP S61-205023; see above), E11 (JP H02-55123) and E12 (JP H03-44627) is also acknowledged in the decision.

Moving on to the assessment of obviousness, the decision emphasizes that it is up to the party that alleges nullity of a patent to make the relevant allegations. The court only examines whether the grounds put forward lead to the invalidity of the patent. Even though obviousness is a question of law, the relevant factual assertions must be made by the party that asserts nullity. In the following, therefore, the decision only deals with the assessment of obviousness starting from document E2 that has been referred to as ‘closest prior art’ by the defendant, i.e. US 4,083,245.

Features f), j) and k) are missing in E2.

With E2 as closest prior art, the decision holds that documents E7 (DE 32 25 114 C1, erroneously referred to as DE 32 25 115 C1 in the decision), E8 (US 1,768,563 A) and E9 (JP H03-21735 U) fail to render the limited claim obvious — mainly because these devices are made from metal / intended for use in high pressure applications, contrary to flow sensors in the medical field which are typically made of plastic, using die casting methods.

In sum, the more limited claim that the plaintiff had submitted with his second reply was thus held to be valid.

Infringement

Defendant most fiercely contested that the adult version of the flow sensors has flange pressure surfaces and, even if there were flange pressure surfaces, that the connections from the connection points to the inside of the first/second half of the housing run partially into or through the flange pressure surfaces. Defendant referred to the computer tomographic images that had been submitted by the plaintiff:

CT image of the flange of an attacked embodiment

The above figure shows a cross-section through the flanges of the two halves of the housing. The flow resistor which is clamped between the flanges is marked with 23. Identifiers 45 and 47 mark a groove and comb structure. Defendant argued that the gas-tight connection is made exclusively via the groove/comb structure. However, the flange surfaces would not touch each other (red arrows in the above figure); only the planes of the comb and groove that protrude perpendicularly from the flange surface would be in contact (white arrows in the above figure). Defendant argued that this is intentional because it allows flow resistors of different thicknesses to be used without the housing halves having to be designed differently, contrary to the teaching of the patent in suit. Since, by definition, flange pressure surfaces would have to lie against each other under pressure, flange pressure surfaces are missing in the adult version, in the defendant’s view. And even if the area in which the flow resistor is trapped by the flanges (marked with a yellow arrow in the figure above) were still called the ‘flange pressure surface’, no connections would occur there in any case. Defendant referred to the below figure in this respect (with the white squares / annotations added by the court):

Flange, annotated by the court

The above figure again shows a cross-section of an adult version of the flow sensor. The thin vertical line is the flow resistor; identifier 33 refers to one of the connections to a connection point which seems to end on the right because it runs diagonally in the Z plane. The decision holds that ‘flange pressure surface’ and ‘flange surface’ are used as synonyms in the patent. A flange pressure surface in the sense of the claim is the surface of a flange facing away from the body part to which the flange is attached and serving to connect the body part to another body part, whereby the connection need not necessarily be made over the entire surface.

Thus, the flange pressure surface of the attacked embodiment runs over the entire width of the flange, i.e. in the above figure from point a) to point b). Consequently, the decision holds that the connections from the connection points to the inside of the first/second housing parts also run through the flange pressure surfaces, as can be seen in the figure above.

The decision holds that the defendant may have improved the teaching of the patent in suit, and the improvement might even be patentable. This does not, however, lead out of the scope of protection of the earlier patent (cf. Art. 36 PatA). In sum, the decision holds that the version for adults is literally covered by the limited claim of the patent. Likewise, the version for children is also held to be literally covered by said claim.

Wording of the injunctive relief

The defendant objected to the wording of the request for injunctive relief, for that it is too indefinite and merely recites the wording of claim features that are under dispute between the parties (such as e.g. the term ‘flange pressure surface’). The decision holds that the features are defined in the reasons of the decision beyond any doubt, so that a mere factual check by the competent authorities is well sufficient. Thus, it is neither necessary nor helpful for each and every feature to be defined in the prayer for relief itself.

Again, this reminds me a bit of the German approach where the mere claim wording is typically used in prayers for injunctive relief, and the exact scope is then to be deduced from the reasons of the decision. I do have some symphathy for the FPC’s approach that keeps the verdict clean and simple, with all the potential ambiguities being resolved by way of definitions given in the reasons.

In sum, the defendant was ordered to abstain from further use of the infringing embodiments, and to provide information and accounting concerning the same.

The parties commented on the decision with the following press releases:

The decision is not yet final; an appeal is pending at the Supreme Court.

Reported by Martin WILMING

BIBLIOGRAPHY

Case No. O2016_009 ¦ Decision of 18 December 2018 ¦ ‘Durchflussmessfühler’

Hamilton Medical AG
./.
imtmedical AG

Panel of Judges:

  • Dr. Mark SCHWEIZER
  • Dr. Tobias BREMI
  • Dr. Lorenzo PARRINI

Judge-rapporteur:

  • Dr. Tobias BREMI

Court Clerk:

  • Susanne ANDERHALDEN

Representative(s) of Plaintiff:

Representative(s) of Defendant:

  • Dr. Rudolf RENTSCH (IPrime)
  • Ernst BREM (IPrime)
  • Paul ROSENICH (PPR)
  • Joachim KÜNSCH (PPR)

DECISION IN FULL

Fullscreen view (new tab)

CH 701 755 B1

Fullscreen view (new tab)

BE ON THE KNOW

Enter your name and email address below to get notified of new posts by email.

The Supreme Court sets the ‘infringement test’ aside — but with a ‘Swiss touch’

Supreme Court
Case No. 4A_576/2017 | Decision of 11 June 2018 | Appeal against decision O2017_001 of 3 October 2017

Note that Hepp Wenger Ryffel is involved in this matter on behalf of the plaintiff.
Gilead's Truvada
Gilead’s Truvada®

Please see this Blog here for a summary of the first instance decision of the FPC.

Nullity of the supplementary protection certificate C00915894 has been at stake. The basic patent is EP 0 915 894 B1; see EPO Register and Swissreg. On a more general level, this case was all about what it needs for a product to be protected by a basic patent, which is a prerequisite for an SPC both in the EU (Regulation (EC) No. 469/2009, Art. 3 lit. a) and Switzerland (Art. 140b(1) lit. a PatA).

It was beyond dispute between the parties that the subject-matter of the SPC (tenofovir disoproxilfumarat + emtricitabin) is covered by the basic patent EP’894. The so-called ‘infringement test’ that had been applied in Switzerland since the Supreme Court’s decision BGE 124 III 375 – Fosinopril in 1998 was thus met. However, the CJEU explicitly disapproved the ‘infringement test’ with its decision CJEU C-322/10 – Medeva of 2011, and the plaintiff argued that the ‘infringement test’ should no longer be applied in Switzerland, either.

The FPC had held that it is not appropriate to change the practice. On the contrary, the Supreme Court did now exactly that.

In first place, the Supreme Court reviewed the practice of the CJEU which initially left it to the national courts to decide on what it meant to be protected by a basic patent. Essentially two lines of jurisprudence developed thereafter, i.e. the disclosure theory (‘Offenbarungstheorie’) and the infringement test (‘Verletzungstest’). Only later, the CJEU disapproved the ‘infringement test’; CJEU C-322/10 – Medeva.

The Supreme Court noted that the Swiss SPC legislation had been enacted with the explicit aim to make it materially the same as in the European Union. The ‘infringement test’ cannot achieve this aim anymore, and it thus cannot be maintained; ¶2.2.5-2.2.6:

Die Auslegung […] weicht konzeptionell ab von der Auslegung durch den EuGH. […] Das vom schweizerischen Gesetzgeber angestrebte Zeil, das Schutzniveau für das Institut der ergänzenden Schutzzertifikate  mit demjenigen im benachbarten Ausland in Einklang zu bringen, wird damit verfehlt. […] An  BGE 124 III 375 kann nicht festgehalten werden.


Noteworthy, the Supreme Court also briefly touched the IPI’s prior initiative to amend its SPC granting practice in light of the CJEU’s Medeva case law, and the positive feedback that had been received from (at least some of) the interested circles. The Supreme Court notes that this was a strong indication that the Swiss practice should indeed be changed, and the IPI’s initiative thus paid off. Still, it was good that the IPI’s initiative had been put on hold in view of the present proceedings. In my humble opinion, any change of practice while BGE 124 III 375 – Fosinopril was still formally applicable would have been premature. Just imagine the consequences if the granting practice had been changed and the Supreme Court later did not abstain from the ‘infringement test'(!), for any reason whatsoever. Dreadful.

Now, what is the test instead? Practitioners are familiar with the subtle twists in the various decisions of the CJEU. This is not further clarified in the present decision. Unsurprisingly, the Swiss Supreme Court essentially only summarizes the criteria of these decisions, ¶2.2.6:

Bezeichnet ein Grundpatent nur einen von zwei Wirkstoffen, kann ein Erzeugnis […] nicht als ergänzendes Schutzzertifikat beanspruch werden, wenn es aus zwei Wirkstoffen zusammengesetzt ist. Art. 140b PatG ist vielmehr […] so auszulegen, dass die Wirkstoffe des Erzeugnisses im Grundpatent beansprucht werden müssen, indem sie in den Patentansprüchen benannt werden, oder indem sich die Patentansprüche – im Lichte der Beschreibung ([…]) ausgelegt – zumindest stillschweigend, aber notwendigerweise auf diese Wirkstoffe beziehen, und zwar in spezifischer Art und Weise.

The ‘Swiss touch’

But the Supreme Court did not just change the practice. It did so with a smooth ‘Swiss touch’: Already granted SPCs shall not be affected by the change of practice. The Supreme Court held that, as a rule, formally final administrative decisions cannot be reconsidered or reversed on the basis of a change in case law. In the Supreme Court’s view, the public interest in equal treatment hardly exists in the context of an SPC, contrary to e.g. in social insurance issues. Apart from the fact that the number of SPCs — irrespective of their economic importance — is rather small, the purpose of granting them is precisely to grant privileges to their owners. If a change of the case law now restricts the conditions for granting SPCs in certain cases, the interests of the other market participants are given a higher weighting and the interests — including public interests in health care — are weighed up differently. However, this change in valuation and consideration of the interests involved does not justify the withdrawal of acquired legal positions, in the Supreme Court’s view; see ¶3.6. No national court in the European Union took this approach when the ‘infringement test’ had been abandoned, to the best of my knowledge.

Now, what is next? Respondent’s counsel already noted on Kluwer Patent Blog that it remains unclear how pending SPC applications shall be dealt with, but they suggested that the infringement test should also apply in these cases. I feel this could well be handled differently. No subjective right has yet been granted in these cases, and I cannot readily see an overriding interest of the applicants to still get SPCs granted contrary to the changed practice. To strike a balance, one might as well just give applicants a chance to amend their pending applications in view of the changed practice instead.

Reported by Martin WILMING

BIBLIOGRAPHY

Supreme Court
Case No. 4A_576/2017 | Decision of 11 June 2018 | Appeal against decision O2017_001 of 3 October 2017

Mepha Pharma AG
./.
Gilead Sciences Inc.

Panel of Judges:

  • Dr. Christina KISS
    • Dr. Kathrin KLETT
    • Dr. Fabienne HOHL
    • Dr. Martha NIQUILLE
    • Marie-Chantal MAY CANELLAS

Court Clerk:

  •  Dr. Matthias LEEMANN

Representative(s) of Plaintiff / Appellant:

Representative(s) of Defendant / Respondent:

  • Dr. Simon HOLZER (MLL)
  • Dr. Kilian SCHÄRLI (MLL)
  • Dr. Michael RITSCHER (MLL)

SUPREME COURT DECISION

Fullscreen view (new tab)

Inofficial English translation, as provided by Defendant’s Counsel on EPLAW Blog:

Fullscreen view (new tab)

FIRST INSTANCE DECISION

Fullscreen view (new tab)

THE BASIC PATENT

Fullscreen view (new tab)

BE ON THE KNOW

Enter your name and email address below to get notified of new posts by email.

Combining a counterclaim with a main claim against a party which hasn’t sued you?

Case No. O2016_016 ¦ Decision of 16 May 2017 ¦ “Einbezug zusätzlicher Parteien mit Widerklage”

HEADNOTE

Art. 64 CPC; Fixation of the parties to the proceedings.

Litispendence results in a fixation of the parties to the proceedings. Further parties cannot be included into pending proceedings – except for an admissible substitution of a party (Art. 83 CPC) or for other reasons provided by law.

The setup of this case is straightforward: A had sued B for nullity of the patent in suit. B lodged a counterclaim for infringement against A with his reply.

In the very same proceedings, B also sued X AG and Y GmbH for infringement. B argued that this voluntary joinder was admissible in view of Art. 71 CPC: the duties of all these parties result from similar circumstances, ie cooperation with respect to the alleged infringement of the patent in suit.

However, the President held that the parties are fixed upon litispendence; Art. 62 and Art. 64 CPC. Exceptions would only be the admissible substitution of a party (Art. 83 CPC) or other reasons provided by law — which have not been applicable here. A voluntary joinder under Art. 71 CPC would have been possible, but only in separate proceedings.

The President thus dismissed the cases against X AG and Y GmbH for lack of a procedural requirement; Art. 59(1) CPCArt. 23(1) lit. a PatCA.

Court fees of CHF 10’000,– have been imposed on B. No party compensation has been awarded: There was no need to even invite X AG and Y GmbH to file reply.

Reported by Martin WILMING

P.S.:
Admittedly, I am frustrated that I have no clue about the parties this time …

BIBLIOGRAPHY 

Case No. O2016_016 ¦ Decision of 16 May 2017 ¦ “Einbezug zusätzlicher Parteien mit Widerklage”

A AG
(Nullity claimant; defendant in countersuit re infringement)

X AG
(Defendant re infringement)

Y GmbH
(Defendant re infringement)

./.

B
(Nullity defendant; counter-claimant re infringement)

Composition of the Board of the FPC:

  • Dr. Dieter BRÄNDLE

Court Clerk:

  • Esther SCHEITLIN

Representative(s) of A AG:

Representative(s) of B:

  • Dr. Simon HOLZER (MLL)
  • Dr. Mark SCHWEIZER (MLL)
  • Dr. Dirk BÜHLER (Maiwald), assisting in patent matters
  • Dr. Andreas LEDL (Maiwald), assisting in patent matters

DECISION IN FULL

Download (PDF, 111KB)

BE ON THE KNOW

You liked this? And you would like to be notified of new posts? Here you go.

Address for service is no ground for recusal (requested too late anyhow)

Case No. O2014_013 ¦ Order of the Adminstrative Board of Judges of 09 June 2016 ¦ “Ausstandsbegehren”


HEADNOTE

Art. 49 CPC; forfeiture of the right to request recusal; substantiation of internal constellation of a group of companies; acting as address of service for the IPI is not a ground of recusal.

‘As soon as it has become aware’ has to be interpreted strictly; in no case the deadline can be longer than 10 days (r. 5, 6).

If the recusal is reasoned based on the relationship between companies of a group of companies, this relationship must be substantiated in detail (r. 7).

The fact that the judge or the judge’s law firm is acting as a purely administrative address of service for the Swiss Federal Institute of Intellectual Property (IPI) is no ground of recusal (r. 7).


I have already reported on the interim measures in this matter on this Blog here and here. Now, we get a glimpse of what is going on in ordinary proceedings between Gilead Pharmasset LLC and AbbVie, Inc. Actually, the interim assessment of the reporting judge is awaited as the next procedural step.

The parties had been informed already on 05 March 2015 that Dr. Tobias BREMI had been appointed as reporting judge. With notification of 16 March 2016, the parties have been informed that the reporting judge will now establish his interim assessment. Only in reply to this notification, the defendant has informed the court that judge BREMI’s firm, Isler & Pedrazzini AG, represents the Swiss parts of the patents EP 2 079 726 (Gilead Connecticut, Inc.), EP 2 297 105 (Gilead Connecticut, Inc. and Genentech, Inc.), EP 2 611 790 (Gilead Connecticut, Inc. and Genentech, Inc.) and EP 2 611 798 (Gilead Connecticut, Inc. and Genentech, Inc). With reference to the decision BGE 139 III 433 of the Supreme Court (reported here), the defendant requested that judge BREMI shall recuse.

In first place, the Administrative Board of Judges assessed the timeliness of the request. A party that wishes to challenge a judge must file the corresponding request as soon as it has become aware of the reason for recusal; Art. 49 CPC. The FPC now held that any delay of more than 10 days is too late. Decisive is the point of time when the party has become aware or should have become aware when handling the matter with all due care. Subsequent to BGE 139 III 433, the FPC has provided detailed guidelines on how to search for potential grounds for recusal, hyperlinked on the website of the FPC where all judges are listed together with their employer in private practice or industry. A brief database search as outlined in these guidelines would immediately have resulted in at least one of the alleged patents. The FPC leaves no doubt that parties are expected to carry out this check immediately when being notified of the involvement of a certain judge:

Die Vornahme solcher Abfragen ist […] bei pflichtgemässer Aufmerksamkeit jeweils durchzuführen, sobald das Bundespatentgericht die Gerichtsbesetzung bekannt gibt.

Therefore, the order holds that the request clearly is not timely filed and any right to request recusal is forfeited.

But even if the request for recusal would have to be dealt with ex officio (as suggested in some literature), the result would be the same. Note that the plaintiff is Gilead Pharmasset, LLC, not Gilead Connecticut, Inc. No schematic approach in the assessment of an appearance of bias can be applied when connected undertakings are involved; see BGE 139 III 433, r. 2.1.6. It rather depends on the actual circumstances. It needs to be individually assessed whether the indirect relationship (via a connected undertaking) is too close, i.e. comparable to a direct relationship. The FPC makes clear that the mere representation before the Swiss patent office (IPI) is — at least in the present matter — limited to provide a Swiss address for service in case of notifications of loss of rights. It is however unclear whether or not any work on the merits of a case would ever be conducted. This cannot establish a ground of recusal right from the outset, unless specific circumstances indicate the contrary (r. 7):

[A]us der blossen Existenz einer solchen Vertretung, ohne Kenntnis der weiteren Umstände, [kann] nicht auf eine Beziehung zur betreffenden Partei geschlossen werden, die einen Ausstandsgrund darstellen könnte.

Endlich kommt dazu, dass sich im vorliegenden Fall die Vertretertätigkeit darauf beschränkt, als administrative Zustelladresse eine allfällige Rechtsverlustmitteilung des Amtes entgegenzunehmen und – unbearbeitet – weiterzuleiten. Ob die Kanzlei danach mit einer aktiven Vertretertätigkeit beauftragt wird, welche dann auch in Rechnung gestellt werden könnte, und welche einen Ausstandsgrund darstellen würde, ist völlig offen. Aber in dieser Situation als potentieller Beauftragter befindet sich eine Patentanwalts- (und auch Rechtsanwalts-) Kanzlei gegenüber allen potentiellen Klienten, sprich gegenüber allen im Patentbereich tätigen Gesellschaften oder Personen, welche als Auftraggeber in Frage kommen. Das kann von vornherein keinen Ausstandsgrund darstellen.

Reported by Martin WILMING

— BIBLIOGRAPHY —

Case No. O2014_013 ¦ Order of the Adminstrative Board of Judges of 09 June 2016 ¦ “Ausstandsbegehren”

Gilead Pharmasset LLC ./. AbbVie Inc.

Subject(s):

  • Request for recusal

Composition of the Administrative Board of Judges:

  • Dr. Dieter BRÄNDLE
  • Dr. Frank SCHNYDER
  • Dr. Ralph SCHLOSSER

Court Clerk:

  • Susanne ANDERHALDEN

Representative(s) of Plaintiff:

Representative(s) of Defendant:

— FULL TEXT OF THE ORDER —

Download (PDF, 256KB)

Some thoughts on the urinal question (O2014_002)

Case No. O2014_002 ¦ Decision of 25 January 2016 ¦ “Urinalventil: Teilweise Gutheissung Patentverletzung, Äquivalenz, Auskunft und Rechnungslegung, Verjährung”

HEADNOTE

Art. 109 PatA; Art. 69 EPC; Protocol on the Interpretation of Article 69 EPC of 05 October 1973 as revised by the Act revising the EPC of 29 November 2000; Art. 66 para. 1 lit. a PatA: imitation (equivalence), accessibility.

The assessment of accessibility is not to be confused with the assessment of inventiveness. Starting point for the assessment of accessibility is not the general state of the art, but the patent in suit. Therefore it is not to be assessed whether the replaced feature is inventive in view of the state of the art. It is rather to be assessed whether, starting from the teaching of the patent in suit, it is evident for the skilled person that the replaced feature has the same effect (r. 6.5.2.4).
(Clarification of the case law with respect to S2013_001, decision dated 21. März 2013)

The decision is a booklet of 82 pages and it will take me some more time to finish a post on all aspects of the decision. The main hearing took place on 14 September 2015; see this Blog here for some background information on the subject-matter at stake. But the highlight for sure is the headnote which is meant to clarify an important issue in the assessment of equivalence, i.e. the second question relating to accessibility (Auffindbarkeit). So I bring forward this part of the story.

The plain wording of the second question outlined in the hn of S2013_001 may well have caused some confusion. It reads as follows:

Werden die ersetzten Merkmale und deren objektiv gleiche Funktion dem Fachmann durch die Lehre des Patentes nahe gelegt?

Literally (and inofficially) translated:

Are the replaced features and their objectively same function rendered obvious by the teaching of the patent to the person of skill in the art?

Obviousness is a word that practitioners readily associate with assessment of inventive step. In any event, the FPC has now clarified that this must not be mixed up. According to the hn, the following is to be assessed instead (in the official English translation of the hn):

Starting from the teaching of the patent in suit, is it evident for the skilled person that the replaced feature has the same effect?

In the German language of the proceedings, the relevant section of the hn reads as follows:

Ist bei ausgetauschten Merkmalen die Gleichwirkung für den Fachmann bei objektiver Betrachtung ausgehend von der Lehre des Patents offensichtlich?

My first reaction was ‘Alright, got it, that’s the 2nd Schneidmesser question as applied in Germany, with mild Swiss tones.’ On a second thought, I do not think it is. It is a Urinal question. The reasons of the decision even make this clearer (r. 6.5.2.4, referred to in the hn; emphasis added):

Zu beurteilen ist, ob, wenn die Merkmale ausgetauscht sind, die Gleichwirkung für den Fachmann bei objektiver Betrachtung unter Berücksichtigung der Lehre des Patents offensichtlich ist.

Inofficially translated into English:

It is to be determined whether, once the features are replaced, the same effect is evident for the person of skill in the art when objectively assessed in consideration of the teaching of the patent.

While the hn requires “starting from the teaching of the patent” (“ausgehend von der Lehre des Patents”), the reasons only take the teaching of the patent into consideration (“unter Berücksichtigung der Lehre des Patents”). But when doing what? In light of the reasons, this test compares two already given situations (one with the replaced/claimed feature and another one with the replacing feature) and only then asks whether the same effect is evident. To the contrary, the wording of the concise, yet finely chiselled second question in Germany is (see Schneidmesser I, BGH X ZR 168/00, r. 3a, emphasis added):

Was the person skilled in the art enabled by his expertise to find the modified means as having the same effect?

Haben seine Fachkenntnisse den Fachmann befähigt, die abgewandelten Mittel als gleichwirkend aufzufinden?

This clearly asks for the capability of the person of skill in the art to actually find the replacing means (in their capacity to have the same effect). Schneidmesser does not make a comparison of two already given scenarios and – only based theron – an assessment of whether or not the same effect is evident. And Prof. Dr. Meier-Beck (a member of the panel of judges that has issued the Schneidmesser decision) emphasizes that this is intentional:

This question is all about excluding those cases in which an inventive step was necessary to find the modified means as having the same effect. If that is the case, i.e. if the means having the same effect were not obvious to the person skilled in the art, they are outside the scope of the patent (BGH Oct. 24, 1986, Case No. X ZR 45/85, (Befestigungsvorrichtung [Fixing Device])). Because what the person skilled in the art is not able to find and to do based on the patent and helped by his or her knowledge in the art, is not to be granted to the patentee (BGH May 15, 1975, Case No. X ZR 35/72, (Ski-Absatzbefestigung [Ski Heel Attachment])). Else, the fundamental bargain underlying most patent regimes is violated — i.e. that a patent is an exclusive right granted for a limited period in exchange for certain technical teaching made available by the patentee to the public, by publication of the patent.

(Meier-Beck in Pumfrey, Nicholas; Adelman, Martin J.; Basheer, Shamnad; Dave, Raj S.; Meier-Beck, Peter; Nagasawa, Yukio; Rospatt, Maximilian; and Sulsky, Martin (2009) “THE DOCTRINE OF EQUIVALENTS IN VARIOUS PATENT REGIMES — DOES ANYBODY HAVE IT RIGHT?”, Yale Journal of Law and Technology: Vol. 11: Iss. 1, Article 9. Available here.)

But let’s again have a look at the reasons of the decision S2013_001, r. 17.2: In light of various Supreme Court decisions (BGE 97 II 85, r. 1; BGE 125 III 29, r. 3b; BGE 115 II 490, r. 2a), it was held that it has to be asked whether the replacing means are rendered obvious by the teaching of the patent for the person of routine skill in art, “henceforth referred to as the second question” (“nachstehend als zweite Frage bezeichnet”). In the next paragraph, the second Schneidmesser question is recited and it was held, that this question equates with the aforementioned second question (“entspricht obiger zweiter Frage”). This is why I felt that the Swiss approach to the second question actually resembles Schneidmesser. With the reasons of the present decision at hand, I doubt that this is the case.

It may well be that the answers to the second Schneidmesser question and the Urinal question are the same in some situations. But in my perception this is not necessarily the case. Assume that the replacing feature is taken from a completely different field of technology. Once it is on the table, it may well be the case that the same effect is evident for the person of routine skill in the art, in consideration of the teaching of the patent. But would he ever have actually found it (not only as having the same effect, but maybe even at all)? I do not think that the Urinal question clearly addresses this. Instead, it seems to me that the Urinal question is more along the lines of the second Improver question in the U.K. ([1990] F.S.R. 181; Chancery Division (Patents Court), 16 May 1989), which reads as follows:

Would this (i.e. that the variant had no material effect) have been obvious at the date of publication of the patent to a reader skilled in the art?

I remain keen to learn how the Urinal question will henceforth be applied in practice.

Reported by Martin WILMING (to be cont’d)

— BIBLIOGRAPHY —

Case No. O2014_002 ¦ Decision of 25 January 2016 ¦ “Urinalventil: Teilweise Gutheissung Patentverletzung, Äquivalenz, Auskunft und Rechnungslegung, Verjährung”

Daspaco AG ./. Von Allmen AG

Board of Judges:

  • Dr. Dieter BRÄNDLE
  • Dr. Tobias BREMI
  • Dr. Christoph GASSER
  • Dr. Ralph SCHLOSSER
  • Werner ROSHARDT

Court Clerk

  • Susanne ANDERHALDEN

Reporting Judge:

  • Dr. Tobias BREMI

Representative(s) of Plaintiff:

Representative(s) of Defendant:

  • Matthias STÄDELI (Rentsch)
  • Dr. Demian STAUBER (Rentsch)
  • Dr. Jens OTTOW (Rentsch), assisting in patent matters
  • Dr. Alfred KÖPF (Rentsch), assisting in patent matters

— FULLTEXT —

Download (PDF, 802KB)

— BE ON THE KNOW —

You liked this? And you would like to be notified of new posts? Here you go.

Nordson ./. GEMA: Claim construction and patentability — which court has it right?

Case No. O2013_008 ¦ Decision of 25 August 2015 ¦ “Gutheissung Patentverletzung, Auskunft und Rechnungslegung (Stufenklage)”

 

HEADNOTE

Art. 66(2) lit. b PatA:
Based on Art. 66(2) lit. b PatA in case the court finds infringement there is a substantive entitlement to information and accounting.

[Reference to al. 2 of Art. 66 PatA in the headnote might be misleading; there is no formal al. (2) in Art. 66 PatA. This is marked-up / corrected in the quotation above.]

 

1.  Background of the case

The main hearing in this infringement case has taken place on 15 June 2015; see this Blog here. The patent in suit is EP 899 016 B1 of Nordson Corp.; see Swissreg for further bibliographic details. It pertains to a spray gun for powder coatings like the one shown in Fig. 2:

Fig. 2 of EP 899 016 B1
Fig. 2 of EP 899 016 B1

The only independent claim 1 as granted reads as follows (structured according to the feature analysis recited in the decision):

O1: An apparatus (20) for use in applying coating material to an object

O2: comprising a spray gun (24) having

O3: a handle portion (26),

O4: an extension portion (28)

O5: which is connected with the handle portion (26),

O6: a nozzle (42)

O7: connected with the extension portion (28),

O8: an electrode (46) disposed adjacent to the nozzle (42)

O10: and away from which electrostatically charged coating material flows toward the object,

O11: a coating material flow control member (74)

O12: connected with the handle portion (26) and

O13: manually operable to an actuated condition to initiate a flow of coating material from a coating material passage (62) in said extension portion (28) through the nozzle (42) toward the object,

characterised in that

K14: the spray gun (24) also has a purge air flow control member (110)

K15: connected with the handle portion (26) and

K16: which is manually operable to initiate a flow of air from the coating material passage (62) in the extension portion (28) through the nozzle (42) to remove coating material from the spray gun (24).

2.  What’s it all about?

The plaintiff markets a product according to the patent under the tradename Encore®; see the so-called PowerPurge trigger on the handle in the snapshot below:

power_purgeThe defendant is GEMA Switzerland GmbH. Defendant’s attacked product is the OptiFlex 2 GM03 gun:

optiflex2_product_figure

The attacked embodiment is further specified in the wording of the judgement with three functional illustrations, as follows (with corresponding annotations in English):

As can be seen in the above illustrations, the attacked embodiment has only one trigger (17) at the front of the handle (the Encore® gun has two triggers there; see above illustration). In normal operation, trigger (17) of the attaecked embodiment triggers the powder flow. By means of the key T3 on the backside remote control of the gun, functionality of the trigger (17) can be switched to activate / stop the rinsing process. The Encore® gun does not have such a dual functionality of one and the same trigger, but rather has separate triggers for powder and purge air.

3.  Claim construction and infringement

Essentially, only three claim features were under dispute with respect to claim construction and infringement.

3.1  Are control members (74) and (110) separate elements or not?

The claim requires a coating material flow control member (74); see feature O11. Feature K14 of the claim further requires that (emphasis added)

the spray gun (24) also has a purge air flow control member (110).

Parties dissented on the question of whether or not the material flow control member (74) and the purge air flow control member (110) are to be distinct elements or whether they could both be embodied by one and the same element. Evidently, in the Encore® gun and in the specific embodiments of the patent, these elements are distinct.

The FPC held that both elements may well be realized according to the patent by one and the same element having two functionalities. It is emphasized that embodiments may not be relied on in order to construe the claim in a more limited way than the claims demand for. The general specification and the claims do not mandatorily require that both elements are distinct. The attacked embodiment was thus held to realise feature K14.

But what about the “also” in K14, you ask? Interesting point. Note that the Dusseldorf Regional Court had come to a different conclusion; see decision 4b O 25/13. The Dusseldorf court relied on the German term “ausserdem” (see the German version of claim 1 of EP 899 016 B1). The term used in the (original) English language of the patent is “also”, and the FPC thus exclusively relied on the term “also” (in the sense of “auch”).

3.2  What does connected with the handle portion (26) actually mean?

Feature K15 requires that the purge air flow control member (110) be connected with the handle portion (26). The FPC held that this wording even allows for an indirect connection of purge air flow control member and handle portion.

One-handed operation
One-handed operation

In a function-oriented construction of this feature, the FPC concluded that the connection has to allow for the purge air flow control member being reachable by a user holding the gun at the handle portion. Thus, the purge air flow control member has to be arranged such that a one-handed operation of the gun is enabled.

From an illustration in marketing material of the defendant, it is evident that this functionality is realised by the attacked embodiment.

3.3  And what is the meaning of to initiate a flow of air?

Feature K16 requires that the purge air flow control member (110) be manually operable to initiate a flow of air from the coating material passage (62) in the extension portion (28) through the nozzle (42) to remove coating material from the spray gun (24). It was a matter of dispute whether this covers embodiments where (i) a flow is only initiated if the purge functionality has been unlocked beforehand (by means of another key); or (ii) the flow is not initiated directly, but rather only indirectly by a downstream controller in response to a keystroke on the purge air flow control member. Both situations were held to be covered by this claim feature, and the attacked embodiment was thus held to realise feature K16.

4.  Patentability

The defendant had argued for nullity as a plea in defense. Allegedly, the claimed subject-matter was not novel over D1. However, D1 is silent about the actual location of the purge air flow control element. The FPC thus held that novelty was given, in line with a decision of the Federal Patent Court of Germany in parallel proceedings (see below). Note that D1 is recited as prior art in the patent in suit (see para. [0001] of EP 899 016 B1) and had already been considered during EPO prosecution).

Defendant also argued that the claimed subject-matter was not novel over D2. This gun has a trigger (36) that allows for a flow of air to be directed through channels (51) to the nozzle.

However, there is no flow of air through the powder path, as essentially required by feature K16. Air in D2 is used to generate a swirl into which the powder is then supplied; the purpose is thus a different one compared to the purge air of the patent in suit. Note that there is disclosure of purge air for the supply channel of the powder in D2, but this one is being operated manually by a valve on a separate control panel (11) on the powder pump, i.e. not on the gun itself. Novelty over D2 was thus acknowledged, too. Again, this is in line with the decision of the Federal Patent Court of Germany in parallel proceedings (see below).

Defendant further alleged a lack of inventive step over D1 in further view of the knowledge of the person of skill in the art, exemplified by D3, D4, D5, D6. Apparently, only the argumentation with respect to the combination with D3 had been elaborated in detail. It was beyond dispute that D1 can reasonably be considered as the closest prior art. As mentioned above, D1 is silent about the actual location of the purge air flow control element. From D1 alone, it cannot be concluded that a location for such a control element on the handle was obvious. Other locations such as illustrated in D2 would also be reasonable. Moreover, a foot pedal, clocked or sensory air pulses, or a control element to be used with the other (free) hand of the operator could be envisaged. Thus, the FPC concluded that D1 in further view of the knowledge of the person of skill in the art does not render the claimed subject-matter obvious. This is where the Federal Patent Court of Germany explicitly concluded differently:

Ein derart häufig und darüber hinaus auch noch leicht zu aktivierender Spülluftstrom vermittelt dem Fachmann jedoch, dass das Spülluftstromregelungselement in seiner unmittelbaren Nähe positioniert sein muss und demzufolge nicht entfernt von der Spritzpistole platziert sein kann, da er die Steuerung der Spülluft zudem individuell und somit manuell vornehmen wird. Da die D1 selbst keine Angaben macht, wo ein derartiges Regelungselement angebracht ist, positioniert der Fachmann dieses in unmittelbarer Nähe des Bedieners, wobei die Spritzpistole selbst als Ort der Positionierung am nächstliegenden erscheint.

The FPC held that the German court erred and had based its conclusion on an undue ex-post-facto analysis: D1 is silent about a location in close proximity, and a remote location is not indictated as disadvantageous. Moreover, not even a manual operation / initiation is mentioned in D1. Finally, the combination of D1 with D3 failed mainly because D3 does not concern a powder spray gun, but rather a hydraulic spray gun for liquid substances. Again, the German Federal Patent Court concluded differently. However, the German court obviously relied on a further document D30 (see p. 22, last paragraph) in addition to D3, but this document was not put on file in the Swiss proceedings, for whatever reason.

Further prior art documents D4, D5 and D6 relate to applicators for more-component reactive compositions instead of powder coatings. The  FPC thus held that the person skilled in the art would not even consider these documents.

5.  Legal aspects and costs

5.1  Plea of free prior art

Defendant also presented a plea of free prior art, interpreted by the FPC as a plea for nullity. However, patentability over the cited prior art was acknowledged; see above. The FPC held that a plea of free prior art could only be considered if an infringement under the doctrine of equivalents were at stake. In case of a literal infringement, this plea is pointless:

Die Einrede des freien Stands der Technik könnte ohnehin nur zum Zug kommen, wenn nicht eine wortsinngemässe Patentverletzung, sondern Äquivalenz zu beurteilen ist.

5.2  Substantive entitlement to information and accounting

This is what the headnote is all about. As outlined above, validity and infringment of the patent had been established, but the plaintiff had not provided further reasoning or evidence with respect to the financial compensation sought for in this action by stages. In a nutshell, the defendant argued that only the quantification is deferred to the second stage of the proceedings, but not the whole burden of proof as to the further legal requirements for financial compensation. For instance, the plaintiff had not presented any allegations as to the bad faith of the defendant — a requirement for recovery of defendant’s profits. With respect to damages for lost profits, the plaintiff had neither argued nor substantiated that he makes any profits with products according to the patent.

The FPC held that the defendant’s substantive duty of disclosure and accounting is directly based on Art. 66 lit. b PatA since an infringement had been established. At the first stage of the proceedings, the plaintiff is not obliged to establish the further legal requirements for recovery of defendant’s profits, or damages.

5.3  Costs

The value in dispute was CHF 2.6 m, and the parties apparently put an enormeous effort into it. Compensation for legal representation in the amount of CHF 80’000,– was awarded to the plaintiff, according to Art. 3 ff. CostR-PatC. Noteworthy, costs awarded for the patent attorney were even higher, i.e. CHF 114’185,64.

6.  Parallel decisions in Germany

With decision of April 29, 2015  the German Federal Patent Court had maintained the patent only in a more limited extent, i.e. as follows (marked-up; see 4 Ni 26/13 for details):

An apparatus (20) for use in applying powder coating material to an object comprising a spray gun (24) having a handle portion (26), an extension portion (28) which is connected with the handle portion (26), a nozzle (42) connected with the extension portion (28), an electrode (46) disposed adjacent to the nozzle (42) and away from which electrostatically charged coating material flows toward the object, a coating material flow control member (74) connected with the handle portion (26) and manually operable to an actuated condition to initiate a flow of coating material from a coating material passage (62) in said extension portion (28) through the nozzle (42) toward the object, characterised in that and a coating material passage (62) in said extension portion (28), characterised in that the apparatus has a valve (94), a conduit (98) and a controller (70) which, simultaneously with opening of a valve (54) and closing of a switch (78), operates the valve (94) to an open condition to enable electrode wash air under pressure from a source (96) of electrode wash air to flow through the conduit (98) to the handle portion (26) and from the handle portion (26) of the spray gun (24) through the extension portion (28) of the spray gun (24), the spray gun (24) also has a purge air flow control member (110) connected with the handle portion (26) and which is manually operable to initiate a flow of air from the coating material passage (62) in the extension portion (28) through the nozzle (42) to remove coating material from the spray gun (24).

These amendments in the German proceedings were not based on dependent claims; the added features were taken from the description. 

In infringement proceedings, the Düsseldorf Regional court decided for non-infringement of the patent as granted, neither literally nor under the doctrine of equivalents; see decision 4b O 25/13.

What’s next?

The decision has been appealed to the Supreme Court. A decision can be expected by mid 2016.

Reported by Ingo LUMMER and Martin WILMING

— BIBLIOGRAPHY —

Case No. O2013_008 ¦ Decision of 25 August 2015 ¦ “Gutheissung Patentverletzung, Auskunft und Rechnungslegung (Stufenklage)”

Nordson Corporation ./. GEMA Switzerland GmbH

Composition of the Board of the FPC:

  • Dr. Dieter BRÄNDLE
  • Dr. Tobias BREMI
  • Dr. Herbert LÄDERACH
  • Christoph MÜLLER
  • Dr. Simon HOLZER

Court Clerk

  • Susanne ANDERHALDEN

Reporting Judge:

  • Dr. Tobias BREMI

Representative(s) of Plaintiff:

  • Dr. Christian HILTI (Rentsch)
  • Dr. Jens OTTOW (Rentsch), assisting in patent matters

Representative(s) of Defendant:

— DECISION IN FULL —

Download (PDF, 746KB)

— BE ON THE KNOW —

You liked this? And you would like to be notified of new posts? Here you go.

No pleadings under the guise of the unconditional right to reply

Case No. O2013_020 ¦ Decision of 29 October 2014 ¦ “Verfügung des Bundespatentgerichts vom 29. Oktober 2014 ¦ Unbedingtes Replikrecht”

HEADNOTE

Art. 6 EMRK; Art. 29(1) BV:
Upholding the unconditional right to reply does not allow the parties to make submissions that are not yet or no longer permitted under CPC.

Further to the decision in the matter O2013_004, this is yet another landmark decision on the unconditional right to reply. In the present matter, the plaintiff had filed a submission in reply to the rejoinder of the defendant. In turn, the defendant had been invited to reply to the new requests, pieces of evidence and allegations brought forward in that submission of the plaintiff. The FPC had explicitly instructed that only comments on new matter were to be made, and that each and every allegedly new item should be identified.

The defendant filed his reply, but the FPC held that it obviously did not comply with the aforementioned requirements. In his summary, the defendant had not even addressed the prior submission of the plaintiff. Only some parts of the submission dealt with allegedly new subject-matter brought forward by the plaintiff. Other parts evidently dealt with subject-matter that had been on file beforehand (“plaintiff again re-iterates”, “plaintiff keeps insisting”), or were mere free pleading.

Regular written proceedings were concluded with the rejoinder of the defendant, with the exception that the right of the parties to be heard must be complied with. This was what the FPC’s invitation of the defendant to file a reply was all about. The FPC took the opportunity to again expand on the unconditional right to reply. Such right of course exists (cf. BGE 138 I 484, r. 2.1), but the court may nevertheless assess the admissibility of any such reply. Any submission that goes beyond the instructions of the court are inadmissible. Parties are barred from further submissions up to the main hearing (Art. 228 CPC), with due consideration of Art. 229 CPC, i.e. that presentation of new facts and evidence at the main hearing is only admissible if they are so-called proper novae or, at least, improper novae.

Attempts to circumvent these rules of procedure under the guise of a submission that was only meant to safeguard the unconditional right to reply are not appreciated by the FPC. Rather, one runs the risk that any such submission is held inadmissible.

The FPC thus set a non-extendable time limit of only a few days for the defendant to improve his submission in order to comply with the initial instructions of the FPC. Otherwise, the submission would be disregarded in toto.

Reported by Martin WILMING

BIBLIOGRAPHY

Case No. O2013_020 ¦ Decision of 29 October 2014 ¦ “Verfügung des Bundespatentgerichts vom 29. Oktober 2014 ¦ Unbedingtes Replikrecht”

(not identified) ./. (not identified)

Subject(s):

  • Nullity

Composition of the Board of the FPC:

  • Dr. Dieter BRÄNDLE (President)
  • Susanne ANDERHALDEN (First Court Clerk)

Representative(s) of Plaintiff:

Representative(s) of Defendant:

  • Dr. Michael RITSCHER (MLL)
  • Dr. Mark SCHWEIZER (MLL)

Full text of the decision right here:

Download (PDF, 88KB)

Large date indication of De Grisogono’s “Instrumento Grande”: Patented, yet held to infringe

De Grisogono: Instrumento Grande (left) and Instrumento Grande Open Date (right)

Case No. O2012_033 ¦ Decision of 30 January 2014 ¦ “Conclusions; fardeau d’allégation; double protection”

HEADNOTE

(Official version in English language)

1.  Art. 72 PatA:
Request for an order to cease and desist a patent infringement.

For a request for an order to cease and desist a patent infringement to be admissible it must contain a detailed description of the incriminated act. This description must be sufficiently specific such that a purely factual examination is sufficient to determine whether an act is prohibited. A description which requires a legal qualification or the interpretation of ambiguous technical expressions is insufficient. A request for an order can therefore be limited to the wording of a patent claim only if the wording of the patent claim itself fulfils these requirements (r. 17).

2.  Art. 26 PatA; Art 8 CC:
Burden of allegation if a party raises a ground for nullity of a patent.

If a party raises a ground for nullity the assessment of which involves the skilled person [and] his common general knowledge, this party bears the corresponding burden of allegation, i.e. it must specifically name the skilled person (defined according to education and/or profession) and his common general knowledge (in particular the extent of the pertinent technical knowledge supposed to be known to him at the relevant date). If such a specific allegation is not put forward the ground for nullity will be dismissed (r. 19-20 and 31-32).

3.  Art. 125 PatA; Art. 51PatA:
Prohibition of double patenting.

The loss of effect of the Swiss patent due to the existence of a European patent for the “same invention” with the same effective date according to Art. 125(1) PatA is subject to the condition that the same technical teaching is protected by the claims of both patents (r. 37).

1.  Procedural History

Richemont International S.A. had sued De Grisogono S.A. for patent infringement before the Civil Court of the canton of Geneva on 15 July 2008. The defendant delivered his answer on 04 November 2008 (the year 2011 given in the decision on p.6 is evidently a typo), argued for non-infringment, lodged a counterclaim for nullity of the patent in suit and requested that the infringemenent action be held an act of unfair competition. As an auxilliary measure, the defendant requested to appoint an expert. With his reply of 30 January 2009, the plaintiff inter alia requested that three experts were to be appointed. The rejoinders were filed on 27 April 2009 (defendant) and 12 July 2009 (plaintiff), respectively.

Finally, three experts were appointed: René BESSON (horologist), Ronald NOLL (patent attorney) and René ADDOR (horologist). BESSON and NOLL submitted a joint opinion on 05 April 2011; ADDOR completed his opinion on 13 June 2011.

The Civil Court of Geneva decided to transfer the case to the Federal Patent Court on 26 January 2012. The parties were invited on 23 October 2012 to comment on the two expert opinions. They filed their comments on 14 January 2013, and the plaintiff put the competence of the FPC into doubt. The main hearing took place on 05 November 2013 (extra muros in Fribourg).

2.  The patent in suit

The patent in suit is CH 695 712 A5 of Richemont International S.A.; for bibliographic details and legal status, see Swissreg. In brief, independent claim 1 pertains to a mechanism to display a date from two separate numbers, comprising

a first annulus (1) with a first set of digits and a toothing at its inner circumference to drive it;

a mobile (9,10) with a plate (9) comprising a second set of digits and a part (10) with a multitude of teeth and wherein the plate (9) is partially arranged on top of the first annulus; such that at each stable position of the first annulus (1) and the mobile (9,10) a digit of the first annulus (1) and a digit of the mobile (9,10) are arranged side by side in a frame or two parts (2,13) of a frame;

characterized in that the mobile (9,10) is pivot-mounted outside of the first annulus (1) and wherein the first annulus (1) comprises a toothing (14) also at its outer circumference, interacting with the teeth of part (10).

For ease of reference, an illustrative figure and the original claim wording in French language is given below:

O2012_033_illustration_patent_in_suit

The mechanism can be best understood on the basis of Fig. 1-3 of CH 695 712 A5 (excerpts of which are given below) showing the sequence of a date switch:

Excerpts from Fig. 1 (left), 2 (middle) and 3 (right) of CH 695 712 A5
Excerpts from Fig. 1-3 of CH 695 712 A5
3.  The incriminated watches

Two de GRISOGONO watches were incriminated, i.e. Instrumento Grande and Instrumento Grande Open Date. A key feature of both these watches is the large date indication (at 6-position in the pictures below):

De Grisogono: Instrumento Grande (left) and Instrumento Grande Open Date (right)
Instrumento Grande (left)
Instrumento Grande Open Date (right)

An annotated picture of one of these watches had been filed by the plaintiff, for illustrative purposes:

instrumento_grande_open_date_-_annotated
Instrumento Grande Open Date, annotated by the plaintiff (taken from p. 5 of the decision).
4.  What has been decided
4.1  Competency of the FPC

According to Art. 41 PatCA, the Federal Patent Court shall, where it is competent, adjudicate in cases that are pending before the cantonal courts, provided that the main hearing has not yet been held. The procedure of taking evidence had not yet been concluded, and no main hearing had taken place. Thus, the cantonal court referred the case to the FPC.

On the subject matter of the case, the competency of the FPC is evident (Art. 26 PatCA).

4.2  Wording of the requests

The wording of the request essentially recites the wording of the only independent claim 1 of the patent in suit. Don’t assume that this is typically the case. Formal requirements for such requests are harsh and sometimes difficult to meet in view of the decision 131 III 70 of the Supreme Court. For a request for an order to cease and desist a patent infringement to be admissible it must contain a detailed description of the incriminated act. This description must be sufficiently specific such that a purely factual examination is sufficient to determine whether an act is prohibited. A description which requires a legal qualification or the interpretation of ambiguous tehcnical expressions is insufficient. A request for an order can therefore be limited to the wording of a patent claim only if the wording of the patent claim itself fulfils these requirements. This was held to be the case here. (But beware: It is more an exception than the rule that recitation of the claim wording is sufficient.)

4.3  On the merits of the patent

Besides the alleged infringment of CH 695 712 A5 (see 4.4, below), the FPC had to deal with validity issues since the defendant raised a counterclaim of invalidity (see 4.3.1 to 4.3.4, below). Note that Swiss patent applications are only formally examined but not on the merits, i.e. for novelty and inventive step. (A pre-examination on the merits had been introduced for certain inventions in the textile and watch industry with an amendment of the PatA in 1954, but this has been abolished again in 1996.)

4.3.1  Alleged undue extension of subject-matter

The amendments carried out during prosecution are discussed in the decision on the basis of the claim wording with tracked changes:

Amendments CH 695 712

The amendments were found to be clearly and unambiguously derivable from the documents as filed. In any case, this would not have been a ground for nullity under Art. 26(1) c PatA, but would rather have resulted in a shifted filing date (Art 58(2) PatA effective until June 30, 2008; BGE 4A_109/2011 / 4A_111/2011, r. 4.1).

As a sidenote, the FPC emphasized that it is the understanding of the person of routine skill in the art that counts for assessment of the question of whether or not an amendment goes beyond of what can be clearly and unambiguosly derived from the application as filed. Thus, the person of routine skill in the art is to be identified beforehand, typically by means of his profession and/or education. Providing this definition is an obligation of the party who raises an argument or counterclaim that relies on the knowledge of the person of routine skill in the art. The defendant did not do so but rather only gave a general definition without any context to the technical matter at stake.

4.3.2  Novelty

The defendant alleged a lack of novelty of the patent in suit over the following documents:

i)  CH 316 461 (Valjoux)

As can be concluded from Fig. 1 of the Valjoux patent (below, for ease of reference), the disk carrying the first series of digits (the upper disk in fig. 1) is not in the form of an annulus (couronne). Consequently, there is also no disclosure of an inner and outer toothing on the (missing) annulus. Novelty over the Valjoux patent was thus acknowledged.

Fig. 1 of CH 316 461 (Valjoux)
Fig. 1 of CH 316 461 (Valjoux)

ii)  CH 689 601 A5 (Piguet)

Piguet discloses an annulus for the first series of digits, but without any outer toothing (but rather only two inner toothings). Moreover, the mobile (carrying the digits 0-3) is not is pivot-mounted outside of the annulus. Novelty over the Piguet patent was thus acknowledged. See Fig. 2 of the Piguet patent below, for ease of reference:

Fig. 2 of CH 689 601 A5 (Piguet)
Fig. 2 of CH 689 601 A5 (Piguet)

iii)  EP 529 191 B1 (Jaeger-Le Coulte)

This piece of prior art was not alleged by the defendant in order to challenge novelty or inventive step (but rather only in the context of the alleged undue extension of subject matter). Nevertheless, ADDOR had discussed this document also for patentability issues — and the plaintiff did not object. Anyhow, novelty over the Jaeger-Le Coultre patent was acknowledged by the FPC: The mobile (carrying the digits 1-3) is not pivot-mounted outside of the annulus. Moreover, there is no outer toothing on the annulus that interacts with the toothing of the mobile. See Fig. 1 of the Jaeger-Le Coultre patent below, for ease of reference:

Fig. 1 of EP 529 191 B1 (Jaeger-Le Coultre)
Fig. 1 of EP 529 191 B1 (Jaeger-Le Coultre)

iv)  JP 44-20619 (Aichi Tokai Denki)

This document had not been relied on by either of the parties. ADDOR had identified it in a search which had not been authorized or instructed by the FPC. Even worse, he came up with this document only two months after he had initially provided his opinion. The plaintiff could have objected against consideration of this document, but did not do so. Anyhow, novelty over the Aichi Tokai Denki patent was acknowledged for essentially the same reasons as outlined for the Valjoux patent above: It does not disclose an annulus being driven by an inner toothing and interacting with a mobile by means of an outer toothing.

Unfortunately, I could not find a pdf of this document, but at least the figure given in the decision is shown below for illustration purposes:

Figure of JP 44-20619 (Aichi Tokai Denki)
Figure of JP 44-20619 (Aichi Tokai Denki)
4.3.3  Inventive step

The FPC assessed the inventive merit on the basis of the so-called problem-and-solution-approach (see EPO Guidelines for Examination, G-VII, 5). The objective technical problem to be solved was identified in a simplification of the mechanism and a reduction of the number of pieces (see paragraphs [0005] and [0006] of CH 695 712 A5). There was no apparent incentive for the person of routine skill in the art to arrive at the claimed invention in an obvious manner; thus, the could-would-approach failed and inventive step was acknowledged.

4.3.4  Double patenting

The plaintiff is also owner of EP 1 296 204 B1. The European application and the Swiss patent CH 695 712 A5 had been filed on one and the same day, with apparently the same documents. However, the resulting scope after prosecution was different in both cases (I did not yet make a detailed comparison of the claim wordings). Thus, the FPC held that both patents are not directed to the same invention and therefore Art. 125 PatA does not apply.

4.3.5  Infringement

The report BESSON/NOLL came to the conclusion that the patent in suit is infringed. From the report ADDOR, the FPC took the same conclusion (albeit implicitly). Taking both reports into due consideration, the FPC followed the conclusions drawn by BESSON/NOLL.

The primary counter-argument of the defendant was that the (let’s call it) structure 3 in the figure below was not an external toothing (“denture périphérique externe” in the original claim wording in French).

Fig. 1 of EP 1 612 628 B1
Fig. 1 of EP 1 612 628 B1 (De Grisogono); this mechanism is embodied in the incriminated watches.

The ADDOR report provided the following illustration of the technical term tooth (dent; left side) and gudgeon (ergot; right side):

Illustrations of technical terms by the expert Addor: dent (left); ergot (right)
Illustrations of technical terms by the expert Addor: dent (left); ergot (right)

The FPC was not at all convinced that the so-called ergot was not a tooth. In fact, even in EP 1 612 628 B1 itself the structure 3 is referred to as a tooth (dent); see paragraph [0010]. The FPC held that also a single tooth constitutes a toothing according to the patent in suit.

5.  In a nutshell

The defendant was ordered to cease and desist from using the invention, and to report on the sales figures of the infringing devices within a deadline of 60 days. The counterclaims of the defendant were dismissed. The decision can be appealed within 30 days.

Reported by Martin WILMING

BIBLIOGRAPHY

Case No. O2012_033 ¦ Decision of 30 January 2014 ¦ “Conclusions; fardeau d’allégation; double protection”

Richemont International S.A. ./. De Grisogono S.A.

Subject(s):

  • Infringement and nullity
  • Unfair competition
  • “Couronne dentée”

Composition of the Board of the FPC:

  • Dr. iur. Dieter BRÄNDLE (President)
  • Dr. Tobias BREMI (Judge)
  • Frank SCHNYDER (Judge)
  • Lic. iur. Jakob ZELLWEGER (First Court Secretary)

Representative(s) of Plaintiff:

  • Maître Michel MUHLSTEIN (JMLP)

Representative(s) of Defendant:

Full text of the decision right here:

Download (PDF, 723KB)

Drospirenone: A masterclass in equivalents

Case No. S2013_001 ¦ Decision of 21 March 2013 (excerpt) ¦ “Auszug aus dem Urteil Nachahmung (Äquivalenz)”

his post has been updated on 02/15/2016 with a more literal translation of the second question, in light of O2014_002.

HEADNOTE

Art. 109 PatA; Art. 69 EPC, Protocol on the Interpretation of Article 69 EPC; Art. 66(1) lit. a PatA:

When an allegedly infringing method or device does not literally fulfil one or more features of the claim but comprises replacing features instead, the following three questions need to be answered in the affirmative in order to conclude that the allegedly infringing method or device is within the scope of the claim:

(1.) Does the replacing feature objectively fulfil the same function? (“Gleichwirkung”)

(2.) Was the person skilled in the art enabled to find the replacing feature as having the same effect? Are the replaced features and their objectively same function rendered obvious by the teaching of the patent to the person of skill in the art? (“Auffindbarkeit”)

(3.) With due consideration of the claim wording in view of the description: Would the person skilled in the art have taken the replacing features into account as being an equivalent solution? (“Gleichwertigkeit”) (r. 17.2)

(Non-official translation from German into English language)

We already catched a glimpse of what was going on earlier this year; see the post on ex-parte injunctions of 26 May 2013. Now, we get some deeper insight into the subject-matter at stake. At first, I felt the take-away message is clear-cut: The FPC assess infringement under the doctrine of equivalents using essentially the Schneidmesser questionnaire established in Germany. That’s it. But this would have been too narrowly considered. On closer scrutiny, things are (much) more difficult. In fact, this case might turn out to be a masterclass in assessment of equivalents in various countries.

1.  Background of the case

Drospirenone
Drospirenone (DRSP; click to enlarge)

The plaintiff sought for preliminary injunctive relief and invoked two European patents: EP 0 918 791 B3 (hereinafter the ‘791 patent; see the European patent register for further details) and EP 1 149 840 B2 (which is a divisional of the ‘791 patent; hereinafter the ‘840 patent); see the European patent register for further details. Both patents relate to methods for the production of drospirenone (DRSP), a compound used in contraceptives (birth control pills).

Drospirenone closely resembles natural progesterone. Drospirenone-containing pills are presumably the most widely prescribed hormonal contraceptives in the world today. Proprietor of both European patents is Bayer Pharma AG, according to Swissreg (cf. here and here). Drospirenone-containing pills are one of the – if not the – best selling product of Bayer Pharma AG; see some further information directly on the website of Bayer Pharma AG. Thus, it’s reasonable to assume that Bayer Pharma AG or a connected undertaking is the plaintiff. The situation is not that clear to me with respect to the defendant(s), and I am not going to speculate.

2. The decision of the FPC

i)  Ex-parte interim measures

As already outlined in an earlier post, the ex-parte injunction in this case was not granted without any prior hearing of the defendant. In fact, the parties had already presented their case in writing, and a hearing with oral reply and rejoinder had already taken place on 31 January 2013. Moreover, the reporting judge had already presented his preliminary opinion on the merits of the case — presumably essentially in the sense of the present decision. Only shortly thereafter, on 7 February 2013, the plaintiff noticed that not all allegedly infringing products were withheld at the customs but that some appeared on the market in Switzerland. Under these circumstances, the FPC granted an ex-parte injunction.

ii)  Rectification of the record

The record of the hearing is quite lengthy (49 p.), and both parties requested rectifications of the record. The plaintiff did so within three days, pointed to five specific issues and indicated precisely what the correction should be. All these rectifications were accordingly made by the FPC.

The defendant filed his request for rectification only after seventeen days. The FPC held that this request was filed too late and thus already failed for this reason. Moreover, the defendant did not indicate what the correction should be, but rather requested the correction to be made on the basis of the tape. The FPC reminded the parties that the record is to contain the overall content of the submissions, but needs not to be a literal transcript (Art. 235(2) CPC; note that the non-official English translation of the CPC fails to properly recite the German wording “dem wesentlichen Inhalt nach“).

iii)  Value in dispute

The plaintiff had estimated the value in dispute to about CHF 500’000,– and indicated that this is a common value in summary proceedings pertaining to patents in the pharaceutical field. On the other hand, the defendant provided a reasoned estimation of the value in dispute to exceed CHF 1’000’000,– . The FPC agreed with the defendant. “Common values” are irrelevant. What counts is the individual assessment of each case.

iv)  Assessment of equivalents

Here we are at the very core of the dispute. A little bit of chemistry is no longer avoidable. Let’s have a look at the final steps in synthesis of drospirenone (DRSP; click to enlarge):

Final steps in synthesis of drospirenone (DRSP)

The starting molecule is the alkine ZK 34506 which is hydrated to form the compound ZK 92836 (step A). Next, ZK 92836 is oxidised to form the cyclic lactone ZK 90965 (step B). Finally, water is eliminated from ZK 90965 to form the enone DRSP (step C).

In simple terms, claim 1 of EP’791 protects the reaction scheme as outlined above, wherein step B is carried out in the presence of a ruthenium salt as a catalyst. Step C is not specified any further.

On the other hand, the only claim 1 of EP’840 specifically pertains to step C only and requires that dehydration is carried out through addition of p-toluenesulfonic acid.

Apparently, the plaintiff sought for injunctive relief with respect to two different methods. With respect to a “first method”, the FPC held that the plaintiff had not credibly shown that this method is effectively used by the (supplier of the) defendant, in particular in view of the injunction issued with decision 4a O 49/12 – Drospirenon of the Dusseldorf regional court; cf. r. 16. This Dusseldorf decision specifically pertains to the dehydration through addition of p-toluenesulfonic acid and refers to WO 2006/061309 A1 in this respect (more specifically to example 7, relating to a method of manufacturing DRSP that does not rely on a ruthenium salt as catalyst in step B, but still uses p-toluenesulfonic acid in step C). Even the plaintiff seemingly suspected that p-toluenesulfonic acid is not used by the defendant (anymore).

The “second method” is the more interesting one. The defendant did not dispute that the “second method” is used. Apparently, the following features were not literally fulfilled:

  • With respect to EP’791, step B is not carried out in the presence of a ruthenium salt as a catalyst, but rather in the presence of TEMPO in combination with Ca(OCl)2.
  • With respect to EP’840, dehydration is not carried out through addition of p-toluenesulfonic acid, but rather in the presence of pyridine/water.

Assessment of infringement of EP’791

The FPC concluded that both the ruthenium salt on the one hand and TEMPO in combination with Ca(OCl)2 on the other hand objectively fulfil the same function (catalysis), in the absence of any further limitation with respect to the specific mechanism of catalysis in the patent in suit (“Gleichwirkung”). The FPC then briefly discussed whether the person skilled in the art was enabled to find TEMPO in combination with Ca(OCl)2 as having the same effect, but finally left this undecided (“Auffindbarkeit”). Rather, the FPC held that the person skilled in the art would not have taken TEMPO in combination with Ca(OCl)2 into account as being an equivalent solution, with due consideration of the claim wording in view of the description (“Gleichwertigkeit”), for the following reasons: First, EP’791 explicitly aims to replace toxic chromium compounds with catalytic amounts of a metal compound. The ruthenium salt catalyis is said to be the key reaction in EP’791 (cf. para.  [0012]). The FPC thus held that this does not suggest the applicability of an organic radical like TEMPO, and EP’791 is not infringed (r. 17.4-17.8).

Apparently, courts in the Netherlands (Rechtsbank Den Haag, C/09/432919 / KG ZA 12-1391), Germany (Dusseldorf regional court, 4a O 190/12 and 4a O 192/12; both confirmed by the higher regional court with decisions 2 U 26/13 and 2 U 25/13, respectively) and Italy (Corto di Appello di Torino) have also issued decisions on this topic. Of course, the final outcome in main proceedings is to be awaited, but this is where we stand today, to the best of my knowledge:

TEMPO / Ca(OCl)2 replacing a ruthenium salt (EP’791) is held to be …
an infringement in … no infringement in …
IT* x
NL x
CH x
DE x
  • From what is discussed in the Dusseldorf decisions, I conclude that the Italian decision relies on an opinion of a court-appointed expert. Unfortunately, I could not yet retrieve a copy of this decision.

Assessment of infringement of EP’840

With p-toluenesulfonic acid, step C of the above reaction scheme is an acid-catalysed elimination of water. On the other hand, pyridine/water is a weakly basic system, relying on a different reaction mechanism. Nevertheless, the result (elimination of water) is the same. The FPC thus held that both reagents fulfill the same function (“Gleichwirkung”). Since both acid- and base-catalysed  elimination reactions are very basic reactions in organic chemistry, the FPC held that the person of skill in the art was enabled to find an appropriate base in general. Moreover, she/he would have also considered pyridine/water in particular since there is a pointer in the introductory part of the patent in suit, i.e. the discussion of the prior art. Therefore, the FPC held that the person of skill in the art was enabled to find pyridine/water as having the same effect (“Auffindbarkeit”). Finally, the FPC held that the person of skill in the art would also have taken pyridine/water into account as an equivalent means, since this reagent is known to her/him (as is evident from the discussion of the prior art in the patent in suit).

Concluding, the FPC held that EP’840 is infringed under the doctrine of equivalents and granted a preliminary injunction.

Again, what’s going on elsewhere? Note that the FPC was aware of the fact that the Rechtsbank Den Haag (C/09/432919 / KG ZA 12-1391) had decided the same question differently (r. 18.3, last para.). Later, the Dusseldorf court in the decisions 4a O 193/12 – Drospirenon II and 4a O 191/12 explicitly took the decision of the FPC into account — and decided for non-infringement. This was confirmed by the Dusseldorf higher regional court in decisions 2 U 23/13 and 2 U 24/13, respectively. To the contrary, the appelate court of Brussels (Belgium) decided for infringement (decision of March 25, 2013 — 2012/KR/127; unfortunately, I could not yet retrieve a copy of this decision). Of course, the final outcome in main proceedings is to be awaited, but this is where we stand today, to the best of my knowledge:

Pyridine/water replacing p-toluenesulfonic acid (EP’840) is held to be …
an infringement in … no infringement in …
NL x
BE x
CH x
DE x

3.  The decision of the Supreme Court

In summary proceedings, the Supreme Court does not examine the case on the merits again. Generally speaking, only violations of the law are reviewed, but not the discretionary adjudication of the first instance. The defendant referred to the decisions of the Dusseldorf regional court; see above. However, this was not considered by the Supreme Court for that it was only lately filed. It would have been interesting to learn more about the Supreme Court’s view on the merits of the case already now, but it’s not much of a surprise that the Supreme Court dismissed the appeal (4A_160/2013).

4.  Remarks

i)  What is not claimed, is disclaimed(?)

Perhaps the best-known statement of the status of the claims in UK law is by Lord Russell of Killowen in Electric and Musical Industries Ltd v Lissen Ltd (1938) 56 RPC 23, 39 (emphasis added):

The function of the claims is to define clearly and with precision the monopoly claimed, so that others may know the exact boundary of the area within which they will be trespassers. Their primary object is to limit and not to extend the monopoly. What is not claimed is disclaimed. The claims must undoubtedly be read as part of the entire document and not as a separate document; but the forbidden field must be found in the language of the claims and not elsewhere.

So, how to deal with the following situation:
The claim requires a feature (A). A variant (B) is outlined in the description of the patent, but not in the claims. Is there any room for equivalency, i.e. to cover embodiments with variant (B) under the doctrine of equivalents?

After the decision Okklusionsvorrichtung of the Federal Supreme Court in Germany, some practitioners feared (and some hoped) that the answer might be a clear “No”. In the meantime, the decision Diglycidylverbindung of the Federal Supreme Court provided some further guidance on the assessment in such cases. In a first step, the specific technical effect of the claimed feature (A) has to be determined in order to differentiate it from the alternative feature (B) mentioned in the description but not included in the claim. Second, it has to be established whether the allegedly infringing means provides for the specific technical effect of the claimed feature (A) and thus differs from the alternative feature (B) mentioned only in the description in the same way as the claimed feature (A).

It will be interesting to read a decision of the FPC in such a situation (in the present matter, the pointer to pyridine/water in the introductory part of EP’840 was apparently not held to preclude equivalency). In an obiter dictum, the FPC seemingly tends to assess such situations in line with the Federal Supreme Court in Germany: It is likely that variants explicitly outlined in the description but not in the claims cannot be considered to be covered under the doctrine of equivalents but rather are waived (r. 17.1, last para.).

Wäre das Mittel nämlich in der Beschreibung aufgeführt, aber nicht im Anspruch, müsste daraus voraussichtlich geschlossen werden, dass der Anmelder auf dessen Beanspruchung verzichtet hat und die betreffende Ausführungsart nicht unter Schutz stellen wollte.

ii)  Questionaires for assessement of equivalents – a broader European picture

In order to assess equivalency, the landmark decision in UK is Improver Corporation v Remington Consumer Products Limited (1990) FSR 181, 189. Questions to be answered in the assessement of equivalency are frequently referred to as Improver questions:

(1) Does the variant have a material effect upon the way the invention works? If yes, the variant is outside the claim. If no-

(2) Would this (i.e. that the variant had no material effect) have been obvious at the date of the publication of the patent to a reader skilled in the art. If no, the variant is outside the claim. If yes –

(3) Would the reader skilled in the art nevertheless have understood from the language of the claim that the patentee intended that strict compliance with the primary meaning was an essential requirement of the invention. If yes, the variant is outside the claim. On the other hand, a negative answer to the last question would lead to the conclusion that the patentee was intending the word or phrase to have not a literal but a figurative meaning (the figure being a form of synecdoche or metonymy) denoting a class of things which included the variant and the literal meaning, the latter meaning, the latter being perhaps the most perfect, best-known or striking example of the class.

In Germany, the Federal Supreme Court (BGH) took over a similar schedule of assessment in the decisions Schneidmesser I and Schneidmesser II:

(1) Does the modified embodiment solve the problem underlying the invention by means which have objectively the same technical effect?

(2) Was the person skilled in the art enabled by his expertise on the priority date to find the modified means as having the same effect?

(3) While answering question two, are the considerations that the person skilled in the art applies drawn from the technical teaching of the patent claim (so that the person skilled in the art took the modified embodiment into account as being an equivalent solution)?

And, finally, the test known from the Formstein decision (X ZR 28/85) of the German Federal Supreme Court (a kind of litmus test to preclude that a claim is construed overly broad, i.e. covering prior art):

(4) Is the modified embodiment anticipated or made obvious by the state of the art?

(A more detailed analysis of the doctrine of equivalents in various jurisdictions can be found in Yale Journal of Law and Technology.)

How was this dealt with in Switzerland by now? Well, questions (1) and (2) were already in use (BGE 97 II 85, r. 2; BGE 97 II 85, r. 1; BGE 125 III 29, r. 3b; BGE 115 II 490, r. 2a.), but a third question was missing — until now. However, the present case strikingly shows that similar questionnaires do not necessarily result in similar decisions. Hopefully, there is no revival of Epilady® or Spannschraube on the rise.

Reported by Martin WILMING


BIBLIOGRAPHY

Case No. S2013_001 ¦ Decision of 21 March 2013 (excerpt) ¦ “Auszug aus dem Urteil Nachahmung (Äquivalenz)”

(not identified) ./. (not identified)

Subject(s):

  • Infringement

Composition of the Board of the FPC:

  • (not identified)

Representative(s) of Plaintiff:

Representative(s) of Defendant:


Full text of the decision right here:

Download (PDF, 128KB)

The mandatory right to be heard, again and again(?)

Case No. O2013_004 ¦ Decision of 15 August 2013 ¦ “Auszug aus dem Entscheid des Präsidenten des Bundespatentgerichts vom 15. August 2013”

HEADNOTE

Art. 6 EMRK; Art. 29(1) and (2) BV:
The parties may be summoned to a hearing to safeguard their mandatory right to be heard and in order to expedite the proceedings.
(Non-official translation from German into English language)

As outlined in an earlier post, parties have a mandatory right to be heard (“unbedingtes Replikrecht”); no time limit needs to be set by the Court, but one has to act quickly (cf. BGE 138 III 252, r. 2.2; BGE 133 I 98, r. 2.2 and a memorandum of the Supreme Court).

But what does that mean in pratice? Can an exchange of replies based on the mandatory right to be heard go on for ever? It can be difficult for a court to come up with a decision when parties don’t stop writing.

In the present matter, the defendant had accepted the plaintiff’s claim on 26 June 2013. The plaintiff filed a submission on 22 July 2013 (inter alia pertaining to the costs). The defendant replied on 09 August 2013. The FPC now took a pragmatic approach to effectively take such a series of replies to an end: The plaintiff was obliged to indicate by no later than 29 August 2013 whether it is intended to again reply. If so, the parties would be summoned to an oral hearing in order to conclude the exchange of positions.

Reported by Martin WILMING

BIBLIOGRAPHY

Case No. O2013_004 ¦ Decision of 15 August 2013 ¦ “Auszug aus dem Entscheid des Präsidenten des Bundespatentgerichts vom 15. August 2013”

(not identified) ./. (not identified)

Subject(s):

  • (not identified)

Composition of the Board of the FPC:

  • Dr. Dieter BRÄNDLE (President)

Representative(s) of Plaintiff:

  • (not identified)

Representative(s) of Defendant:

  • (not identified)

Full text of the decision right here:

Download (PDF, 11KB)